You are on page 1of 97

KENDRIYA VIDYALAYA GACHIBOWLI, GPRA CAMPUS, HYD-32

SAMPLE PAPER TEST 01 FOR BOARD EXAM 2023


(ANWERS)
SUBJECT: MATHEMATICS MAX. MARKS : 80
CLASS : X DURATION : 3 HRS
General Instruction:
1. This Question Paper has 5 Sections A-E.
2. Section A has 20 MCQs carrying 1 mark each.
3. Section B has 5 questions carrying 02 marks each.
4. Section C has 6 questions carrying 03 marks each.
5. Section D has 4 questions carrying 05 marks each.
6. Section E has 3 case based integrated units of assessment (04 marks each) with sub-parts of the
values of 1, 1 and 2 marks each respectively.
7. All Questions are compulsory. However, an internal choice in 2 Qs of 5 marks, 2 Qs of 3 marks
and 2 Questions of 2 marks has been provided. An internal choice has been provided in the
2marks questions of Section E
8. Draw neat figures wherever required. Take π =22/7 wherever required if not stated.
SECTION – A
Questions 1 to 20 carry 1 mark each.
1. The LCM of two numbers is 14 times their HCF. The sum of LCM and HCF is 600. If one
number is 280, then the other number is
(a) 20 (b) 28 (c) 60 (d) 80
Ans: (d) According to the question, LCM + HCF = 600
Since LCM = 14 × HCF
⇒ 14 × HCF + HCF = 600 ⇒ 15 × HCF = 600 ⇒ HCF = 600 ÷ 15 = 40
⇒ LCM = 600 – HCF = 600 – 40 = 560
We know that HCF (a, b) × LCM (a, b) = a × b
⇒ Other number = 40 x 560 /280 = 80

2. When 2120 is expressed as the product of its prime factors we get


(a) 2 × 5³ × 53 (b) 2³ × 5 × 53 (c) 5 × 7² × 31 (d) 5² × 7 × 33
3
Ans: (b) 2120 = 2 × 2 × 2 × 5 × 53 = 2 × 5 × 53

3. If p and q are the zeroes of the quadratic polynomial f(x) = 2x2 – 7x + 3, find the value of p + q
– pq is
(a) 1 (b) 2 (c) 3 (d) None of these
Ans: (b) 2
4. The pair of linear equations 2x + 3y = 5 and 4x + 6y = 10 is
(a) inconsistent (b) consistent (c) dependent consistent (d) none of these
Ans: (c)
a1 2 1 b1 3 1 c1 5 1
  ,   ,  
a2 4 2 b2 6 2 c2 10 2
a1 b1 c1
  
a2 b2 c2
Therefore, the pair of linear equations has infinity many solutions and hence dependent
consistent

5. Points A(3, 1), B(5, 1), C(a, b) and D(4, 3) are vertices of a parallelogram ABCD. The values of
a and b are respectively
(a) a = 6, b = 3 (b) a = 2, b = 1 (c) a = 4, b = 2 (d) None of these
Ans: (a) a = 6, b = 3
Prepared by: M. S. KumarSwamy, TGT(Maths) Page - 1 -
6. If ∆ABC ∼ ∆DEF, BC = 3 cm, EF = 4 cm and area of ∆ABC = 54 cm2. Then the area of ∆DEF
is

(a) 54 cm2 (b) 88 cm2 (c) 96 cm2 (d) 108 cm2


Ans: (c) 96 cm2

7. If sec A = 15/7 and A + B = 90°, find the value of cosec B.


(a) 8/7 (b) 12/7 (c) 7/15 (d) 15/7
Ans: (d)

8. In ΔABC, right angled at B, AB = 5 cm and sin C = 1/2. Determine the length of side AC.
(a) 10 cm (b) 15 cm (c) 20 cm (d) none of these
Ans: (a)

AB 1 5
sin C     AC  10cm
AC 2 AC

9. In the ∆ABC, D and E are points on side AB and AC respectively such that DE || BC. If AE = 2
cm, AD = 3 cm and BD = 4.5 cm, then CE equals

(a) 1 cm (b) 2 cm (c) 3 cm (d) 4 cm


Ans: (c) 3 cm
AD AE 3 2
    CE  3cm
BD CE 4.5 CE

10. ABCD is a trapezium with AD ∥ BC and AD = 4cm. If the diagonals AC and BD intersect each
other at O such that AO/OC = DO/OB =1/2, then BC =
(a) 6cm (b) 7cm (c) 8cm (d) 9cm
Ans: (c) 8cm

Prepared by: M. S. KumarSwamy, TGT(Maths) Page - 2 -


11. If the angle between two radii of a circle is 140°, then the angle between the tangents at the ends
of the radii is
(a) 90° (b) 50° (c) 70° (d) 40°
Ans: (d) 40°

12. The number of revolutions made by a circular wheel of radius 0.7 m in rolling a distance of 176
m is
(a) 22 (b) 24 (c) 75 (d) 40
Ans: (d) 40

13. The area of the square that can be inscribed in a circle of radius 8 cm is
(a) 256 cm² (b) 128 cm² (c) 64√2 cm² (d) 64 cm²
Ans: (b) 128 cm²

14. The ratio of the total surface area to the lateral surface area of a cylinder with base radius 80 cm
and height 20 cm is
(a) 1 : 2 (b) 2 : 1 (c) 3 : 1 (d) 5 : 1
Ans: (d) 5 : 1

15. The mean and mode of a frequency distribution are 28 and 16 respectively. The median is
(a) 22 (b) 23.5 (c) 24 (d) 24.5
Ans: (c) 24
We know that, Mode = 3 Median – 2 Mean
⇒ 3 Median = Mode + 2 Mean
⇒ 3 Median = 16 + 2 × 28 ⇒ Median = 72/3 = 24

16. The median class of the following data is:


Marks 0 – 10 10 – 20 20 – 30 30 – 40 40 – 50 50 – 60
No. of students 8 10 12 22 30 18
(a) 20 – 30 (b) 30 – 40 (c) 40 – 50 (d) 50 – 60
Ans: (b)

17. Two dice are thrown simultaneously. What is the probability of getting doublet?
(a) 1/36 (b) 1/6 (c) 5/6 (d) 11/36
Ans: (b) 1/6
Number of Possible outcomes are 36
Number of favourable outcomes = 6
Probability = 6/36 = 1/6

4sin   cos 
18. If 4 tan  = 3, then the value of is
4sin   cos 
(a) 1/2 (b) 1/3 (c) 1/4 (d) 1/5
Ans: (a) 1/2
Dividing Numerator and Denominator by cos, we get
Prepared by: M. S. KumarSwamy, TGT(Maths) Page - 3 -
3
4  1 3 1 2 1
4sin   cos  4 tan   1 4
    
4sin   cos  4 tan   1 4  3  1 3  1 4 2
4
DIRECTION: In the question number 19 and 20, a statement of Assertion (A) is followed by a
statement of Reason (R).
Choose the correct option
19. Statement A (Assertion): The number 6n never end with digit 0 for any natural number n..
Statement R( Reason) : The number 9n never end with digit 0 for any natural number n.
(a) Both assertion (A) and reason (R) are true and reason (R) is the correct explanation of
assertion (A)
(b) Both assertion (A) and reason (R) are true and reason (R) is not the correct explanation of
assertion (A)
(c) Assertion (A) is true but reason (R) is false.
(d) Assertion (A) is false but reason (R) is true.
Ans: (b) Both assertion (A) and reason (R) are true and reason (R) is not the correct explanation
of assertion (A)

20. Statement A (Assertion): The value of y is 6, for which the distance between the points P(2, –
3) and Q(10, y) is 10.
Statement R( Reason) : Distance between two given points A (x1, y1) and B (x2, y2) is given by
AB = ( x2  x1 ) 2  ( y2  y1 )2
(a) Both assertion (A) and reason (R) are true and reason (R) is the correct explanation of
assertion (A)
(b) Both assertion (A) and reason (R) are true and reason (R) is not the correct explanation of
assertion (A)
(c) Assertion (A) is true but reason (R) is false.
(d) Assertion (A) is false but reason (R) is true.
Ans: (d) Assertion (A) is false but reason (R) is true.

SECTION – B
Questions 21 to 25 carry 2 marks each.

21. In figure, ABCD is a rectangle. Find the values of x and y.

Ans: AB = DC and BC = AD
⇒ x + y = 30 ...(i)
and x – y = 14 ...(ii)
Adding (i) and (ii), we get 2x = 44 ⇒ x = 22
⇒ y = 30 - 22 = 8
Thus, x = 22 and y = 8

22. In the given figure, find the value of x in terms of a, b and c.

Ans: In ∆s LMK and PNK, we have


Prepared by: M. S. KumarSwamy, TGT(Maths) Page - 4 -
∠M = ∠N = 50⁰ and ∠K = ∠K
So, by AA similarity criterion, ∆LMK ~ ∆PNK
LM KM a bc ac
Thus,    x
PN KN x c bc

23. XY and MN are the tangents drawn at the end points of the diameter DE of the circle with
centre O. Prove that XY || MN.
Ans: Since, XY is the tangent to the circle at the point D.
⇒ OD ⊥ XY ⇒ ∠EDX = 90°
Also, MN is the tangent to the circle at E.
⇒ OE ⊥ MN ⇒ ∠ DEN = 90°
As, ∠EDX = ∠DEN (each 90°)
which are alternate interior angles.
⇒ XY ||MN

24. A rope by which a cow is tethered is increased from 16mto 23m. How much additional ground
does it have now to graze?
Ans: Given : length of rope (r) = 16 m
Increased length of rope (R) = 23 m
Hence the additional area cow can graze =  R 2   r 2   ( R 2  r 2 )
22 22
 (232  162 )  (529  256)
7 7
22
  273  858m 2
7
OR
In the below figure, OACB is a quadrant of a circle with centre O and radius 3.5 cm. If OD = 2
cm, find the area of the (i) quadrant OACB, (ii) shaded region.

1 2 1 22 7 7 77
Ans: (i) Area of the quadrant OACB =  r =    = =9.625 cm2
4 4 7 2 2 8
(ii) Area of the BOD = (1/2) x OB x OD = (1/2) x 3.5 x 2 = 3.5 cm2
Area of the shaded region = Area of the quadrant OACB – Area of the BOD
= 9.625 – 3.5 = 6.125 cm2

25. Find A and B, if sin (A + 2B) = √3/2 and cos (A + B) = 1/2.


Ans: Given : sin (A + 2B) = sin 60°
⇒ A + 2B = 60° ...(i)
cos (A + B) = cos 60°
⇒ A + B = 60° ...(ii)
Subtracting equation (i) and (ii), we get B = 0°
Putting the value of B in equation (ii), we get,
A = 60° – 0° = 60°
So, A = 60° and B = 0°.
OR
If (1 + cos A) (1 – cos A) = 3/4 , find the value of tan A.
Ans: (1 + cos A) (1 – cos A) = 3/4
Prepared by: M. S. KumarSwamy, TGT(Maths) Page - 5 -
⇒ 1 – cos²A = 3/4 ⇒ cos²A = 1 – 3/4 = 1/4 ⇒ cosA = ±1/2
Also, 1 – cos²A = 3/4 ⇒ sin²A = 3/4 ⇒ sinA = ±√3/2
⇒ tanA = sinA/cosA = ±√3

SECTION – C
Questions 13 to 22 carry 3 marks each.
26. Prove that √5 is and irrational number.
p
Ans: Let 5 is a rational number then we have 5 , where p and q are co-primes.
q
 p  5q
Squaring both sides, we get p 2  5q 2
 p2 is divisible by 5  p is also divisible by 5
So, assume p = 5m where m is any integer.
Squaring both sides, we get p2 = 25m2
But p 2  5q 2
Therefore, 5q2 = 25m2  q2 = 5m2
 q2 is divisible by 5  q is also divisible by 5
From above we conclude that p and q have one common factor i.e. 5 which contradicts that p
and q are co-primes.
Therefore, our assumption is wrong.
Hence, 5 is an irrational number.

27. Find the zeroes of the quadratic polynomial 6x2 – 7x – 3 and verify the relationship between the
zeroes and the coefficients of the polynomial.
Ans: 6x2 – 7x – 3 = 0
6x2 – 9x + 2x – 3 = 0
3x(2x – 3) + 1(2x – 3) = 0
 (3x + 1) (2x – 3) = 0
1 3
x = ,
3 2
1 3 2  9 7 b 7 b
Now,        and     
3 2 6 6 a 6 a
1 3 1 c 1 c
    and    
3 2 2 a 2 a
28. A part of monthly hostel charges in a college is fixed and the remaining depends on the number
of days one has taken food in the mess. When a student ‘A’ takes food for 22 days, he has to
pay Rs. 1380 as hostel charges; whereas a student ‘B’, who takes food for 28 days, pays Rs.
1680 as hostel charges. Find the fixed charges and the cost of food per day.
Ans: Let the fixed hostel charges be Rs. x and the cost of food per day be Rs. y.
According to the question, we get
x + 22y = 1380 ...(i)
and x + 28y = 1680 ...(ii)
Subtracting (i) from (ii), we get
6y = 300 ⇒ y = 300 ÷ 6 = 50
Putting y = 50 in (i), we get
x + 22(50) = 1380 ⇒ x + 1100 = 1380 ⇒ x = 280
∴ Fixed hostel charges = Rs. 280 and cost of the food per day = Rs. 50.
OR
The ratio of income of two persons is 9 : 7 and the ratio of their expenditure is 4 : 3, if each of
them manage to save Rs. 2000/month. Find their monthly incomes.

Prepared by: M. S. KumarSwamy, TGT(Maths) Page - 6 -


Ans: Let the income of first person be 9x and the income of second person be 7x. Further, let
the expenditures of first and second persons be 4y and 3y respectively. Then, Saving of the first
person = 9x – 4y
Saving of the second person = 7x – 3y
According to question,
9x – 4y = 2000 or 9x – 4y – 2000 = 0 ...(i)
and 7x – 3y = 2000 or 7x – 3y – 2000 = 0 ...(ii)
Solving (i) and (ii), we get x = 2000 and y = 4000
Thus, monthly income of first person = 9 × 2000 = Rs. 18000
Monthly income of second person = 7 × 4000 = Rs. 28000

sin   cos   1
29. Prove that:  sec   tan 
sin   cos   1
tan   1  sec 
And: LHS = tan   1  sec  (Dividing numerator and denominator by cos)

tan   sec   1

tan   1  sec 
tan   sec   (sec 2   tan 2  )

tan   1  sec 
(sec   tan  )(1  sec   tan  )

tan   1  sec 
 sec   tan  = RHS

30. Prove that the intercept of a tangent between two parallel tangents to a circle subtends a right
angle at the center
Ans: Given: XY and X'Y' are two parallel tangents to the circle wth centre O and AB is the
tangent at the point C, which intersects XY at A and X'Y' at B.

In ΔOAP and ΔOAC


AP = AC ( Tangents from to same point A)
PO = OC ( Radii of the same circle)
OA = OA ( Common side)
so, ΔOAP = ΔOAC (SSS congruence criterion)
∴ ∠AOP =∠AOC = ∠1 (CPCT)
Similarly, ∠BOQ =∠BOC = ∠2
Now, POQ is a diameter of the circle.
Hence, it is a straight line.
∴ ∠1 + ∠1 + ∠2 + ∠2 = 180°
2(∠1 + ∠2) = 180°
∴ ∠1 + ∠2 = 90°
∴ ∠AOB = 90°.
OR
Prove that opposite sides of a quadrilateral circumscribing a circle subtend supplementary
angles at the centre of the circle.

Prepared by: M. S. KumarSwamy, TGT(Maths) Page - 7 -


Ans: Let ABCD be the quadrilateral circumscribing a circle at the center O such that it touches
the circle at the point P,Q,R,S. Let join the vertices of the quadrilateral ABCD to the center of
the circle

In ΔOAP and ΔOAS


AP=AS ( Tangents from to same point A)
PO=OS ( Radii of the same circle)
OA=OA ( Common side)
so, ΔOAP=ΔOAS (SSS congruence criterion)
∴ ∠POA=∠AOS (CPCT)
⇒ ∠1=∠8
Similarly, ∠2=∠3, ∠4=∠5 and ∠6=∠7
∠1+∠2 +∠3+∠4+∠5+∠6+∠7+∠8 = 360⁰
⇒ (∠1 +∠8) +(∠2 +∠3) + (∠4 +∠5) + (∠6 +∠7) = 360⁰
⇒ 2(∠1) + 2(∠2) + 2(∠5) + 2(∠6) = 360⁰
⇒ (∠1) + (∠2) + (∠5) + (∠6) = 180⁰
∴ ∠AOD + ∠COD=180⁰
Similarly, ∠BOC + ∠DOA = 180⁰

31. One card is drawn at random from a well-shuffled deck of 52 playing cards. Find the probability
that the card drawn is (i) either a red card or a king, (ii) neither a red card nor a queen.
Ans: Total number of cards = 52
(i) Number of either red card or a king card = 28
28 7
Required Probability = 
52 13
(ii) Number of cards neither a red card or a queen card = 52 – 28 = 24
24 6
Required Probability = 
52 13

SECTION – D
Questions 32 to 35 carry 5 marks each.
3
32. Two water taps together can fill a tank in 9 hours. The tap of larger diameter takes 10 hours
8
less than the smaller one to fill the tank separately. Find the time in which each tap can
separately fill the tank. (NCERT Exercise 4.3 Q9)
OR
A rectangular park is to be designed whose breadth is 3 m less than its length. Its area is to be 4
square metres more than the area of a park that has already been made in the shape of an
isosceles triangle with its base as the breadth of the rectangular park and of altitude 12 m. Find
its length and breadth. NCERT Quadratic Equations Example-12, page no. 84

33. Prove that if a line is drawn parallel to one side of a triangle intersecting the other two sides in
distinct points, then the other two sides are divided in the same ratio.
Using the above theorem prove that a line through the point of intersection of the diagonals and
parallel to the base of the trapezium divides the non parallel sides in the same ratio.

Prepared by: M. S. KumarSwamy, TGT(Maths) Page - 8 -


Ans: For the Theorem :
Given, To prove, Construction and figure of 1½ marks
Proof of 1½ marks
Let ABCD be a trapezium DC ∥ AB and EF is a line parallel to AB and hence to DC.
Join AC, meeting EF in G.

34. A vessel is in the form of an inverted cone. Its height is 8 cm and the radius of its top which is
open, is 5 cm. It is filled with water upto the brim. When lead shots, each of which is a sphere of
radius 0.5 cm are dropped into the vessel, one-fourth of water flows out. Find the number of
lead shots dropped into the vessel.
Ans: We have, height of the conical vessel, h = 8 cm
and radius of the conical vessel, r = 5 cm
1 1 200
∴ Volume of water filled in the vessel cone =  r 2 h    52  8   cm3
3 3 3
Also, we have radius of a spherical lead shot = 0.5 cm
4 4
∴ Volume of each lead shot =  r 3    (0.5)3 cm3
3 3
1 200 50
∴ Volume of lead shots dropped = Volume of water that overflows =   cm3   cm3
4 3 3
50

∴ Number of lead shots dropped = 3  100
4
  0.5  0.5  0.5
3
Hence required number of lead shots is 100.
OR
A copper wire of diameter 8 mm is evenly wrapped on a cylinder of length 24 cm and diameter
49 cm to cover the whole surface. Find (i) the length of the wire (ii) the volume of the wire.
Ans: The thickness of wire = its diameter = 8 mm = 0.8 cm.
And, the length of the cylinder = 24 cm
∴ Number of turns of the wire required to cover the whole surface of the cylinder
= Length of the cylinder/Diameter of the wire = 24/0.8 = 30
Since, diameter of the cylinder = 49 cm
∴ Radius of the cylinder, r = 49/2 cm
(i) Length of wire wrapped in 1 round = Circumference of the cylinder
22 49
= 2 r  2    154cm
7 2
Length of wire wrapped in 30 rounds = 30 × 154 cm = 4620 cm
0.8
(ii) Since radius (r) of wire =  0.4cm and its length or height (h) = 4620 cm
2
22
∴ Volume of the wire =  r 2 h   (0.4)2  4620  2323.2cm3
7
35. If the median of the distribution given below is 28.5, find the values of x and y.
Class 0-10 10-20 20-30 30-40 40-50 50-60 Total
Frequency 5 x 20 15 y 5 60

Prepared by: M. S. KumarSwamy, TGT(Maths) Page - 9 -


Ans: Here, median = 28.5, n = 60

SECTION – E(Case Study Based Questions)


Questions 35 to 37 carry 4 marks each.
36. Anita’s mother start a new shoe shop. To display the shoes, she put 3 pairs of shoes in 1st row, 5
pairs in 2nd row, 7 pairs in 3rd row and so on.

On the basis of above information, answer the following questions.


(i) If she puts a total of 120 pairs of shoes, then find the number of rows required.
(ii) What is the difference of pairs of shoes in 17th row and 10th row.
Ans: Number of pairs of shoes in 1st, 2nd, 3rd row, ... are 3, 5, 7, ...
So, it forms an A.P. with first term a = 3, d = 5 – 3 = 2
(i) Let n be the number of rows required.
∴ Sn = 120 ⇒ (n/2) [2(3) + (n − 1)2] = 120
⇒ n2 + 2n – 120 = 0 ⇒ n2 + 12n – 10n – 120 = 0
⇒ (n + 12) (n – 10) = 0 ⇒ n = 10
So, 10 rows required to put 120 pairs.
(ii) No. of pairs in 17th row = a17 = 3 + 16(2) = 35
No. of pairs in 10th row = a10 = 3 + 9(2) = 21
∴ Required difference = 35 – 21 = 14

Prepared by: M. S. KumarSwamy, TGT(Maths) Page - 10 -


37. In the sport of cricket the Captain sets the field according to a plan. He instructs the players to
take a position at a particular place. There are two reasons to set a cricket field—to take wickets
and to stop runs being scored.
The following graph shows the position of players during a cricket match.
(i) Find the coordinate of the point on y-axis which are equidistant from the points representing
the players at Cover P(2, –5) and Mid-wicket Q(–2, 9)
(ii) Find the ratio in which x-axis divides the line segment joining the points Extra Cover S(3, –
3) and Fine Leg (–2, 7).

Ans: (i) Let A (0, y) be any point on the y-axis.


Since A (0, y) is equidistant from P (2, –5) and Q (–2, 9)
So AP = AQ ⇒ AP2 = AQ2
⇒ (2)2 + (y + 5)2 = (2)2 + (y – 9)2 ⇒ y2 + 10 y + 25 = y2 – 18y + 81
⇒ 28y = 81 – 25 ⇒ 28y = 56
⇒ y = 28/56 = 2
So, the point is (0, 2)
(ii) Let point P(x, 0) divides the line segment joining the points A and B in the ratio k : 1

Hence, the point P divides the line segment in the ratio 3 : 7.

38. A 1.2 m tall girl spots a balloon moving with the wind in a horizontal line at a height of 88.2 m
from the ground. The angle of elevation of the balloon from the eyes of the girl at any instant is
60°. After 30 seconds, the angle of elevation reduces to 30° (see the below figure).

Prepared by: M. S. KumarSwamy, TGT(Maths) Page - 11 -


Based on the above information, answer the following questions. (Take √3 =1.732)
(i) Find the distance travelled by the balloon during the interval.
(ii) Find the speed of the balloon.
Ans: (i) In the figure, let C be the position of the observer (the girl).
A and P are two positions of the balloon.
CD is the horizontal line from the eyes of the (observer) girl.
Here PD = AB = 88.2 m − 1.2 m = 87 m

Thus, the required distance between the two positions of the balloon = 58 √3 m
= 58 x 1.73 = 100.46 m (approx.)
(ii) Speed of the balloon = Distance/time = 100.46/30 = 3.35 m/s (approx.)

Prepared by: M. S. KumarSwamy, TGT(Maths) Page - 12 -


KENDRIYA VIDYALAYA GACHIBOWLI, GPRA CAMPUS, HYD-32
SAMPLE PAPER TEST 02 FOR BOARD EXAM 2023
(ANWERS)

SUBJECT: MATHEMATICS MAX. MARKS : 80


CLASS : X DURATION : 3 HRS
General Instruction:
1. This Question Paper has 5 Sections A-E.
2. Section A has 20 MCQs carrying 1 mark each.
3. Section B has 5 questions carrying 02 marks each.
4. Section C has 6 questions carrying 03 marks each.
5. Section D has 4 questions carrying 05 marks each.
6. Section E has 3 case based integrated units of assessment (04 marks each) with sub-parts of the
values of 1, 1 and 2 marks each respectively.
7. All Questions are compulsory. However, an internal choice in 2 Qs of 5 marks, 2 Qs of 3 marks
and 2 Questions of 2 marks has been provided. An internal choice has been provided in the
2marks questions of Section E
8. Draw neat figures wherever required. Take π =22/7 wherever required if not stated.
SECTION – A
Questions 1 to 20 carry 1 mark each.
1. The value of ‘a’, if HCF (a, 18) = 2 and LCM (a, 18) = 36, is: (1)
(a) 2 (b) 5 (c) 7 (d) 4
Ans: (d) 4
Given, HCF (a, 18) = 2 and LCM (a, 18) = 36
HCF (a, b) × LCM (a, b) = a × b
⇒ 2 × 36 = a × 18 ⇒ a = 4
Hence, value of ‘a’ is 4.

2. The LCM of smallest two-digit composite number and smallest composite number is:
(a) 12 (b) 4 (c) 20 (d) 44
Ans: (c) 20
Smallest 2-digit number = 10 = 2 x 5
Smallest composite number = 4 = 22
LCM = 22 x 5 = 20

3. If r = 3 is a root of quadratic equation kr2 – kr – 3 = 0, then the value of k is:


(a) 1/2 (b) 3 (c) 1/3 (d) 1/4
Ans: (a) 12
Given, equation is kr2 – kr – 3 = 0
If, r = 3, then, k(3)2 – k(3) – 3 = 0
⇒ 9k – 3k – 3 = 0 ⇒ 6k = 3 ⇒k=½

4. The solution of the following pair of equation is:


x – 3y = 2, 3x – y = 14
(a) x = 5, y = 1 (b) x = 2, y = 3 (c) x = 1, y = 2 (d) x = 1, y = 4
Ans: (a) x = 5, y = 1
Given, equations are x – 3y = 2 …(i)
and 3x – y = 14 …(ii)
Solving equations (i) and (ii), we get y = 1
x = 2 + 3y = 2 + 3 × 1 = 5
Hence, x = 5 and y = 1
Prepared by: M. S. KumarSwamy, TGT(Maths) Page - 1 -
5. What is the positive real root of 64x2 – 1 = 0?
(a) 1/8 (b) 1/4 (c) 1/2 (d) 1/6
Answer: (a) 1/8,−1/8
Given, 64x2 – 1 = 0
⇒ (8x + 1) (8x – 1) = 0
⇒ 8x = – 1, 8x = 1
⇒ x = 1/8,−1/8
Thus, positive root is 18

OA OC
6. In the figure, if  , then
OD OB

which pair of angles are equal? [1]


(a) ∠A = ∠C, ∠B = ∠D (b) ∠A = ∠B, ∠C = ∠D
(c) ∠C = ∠B, ∠A = ∠D (d) None of these
Ans: (a) ∠A = ∠C, ∠B = ∠D
OA OC OA OD
In given figure,   
OD OB OC OB
Then, in ΔAOD and ΔCOB
OA OD
 (proved above)
OC OB
and ∠AOD = ∠COB (vertically opposite angles)
∴ ΔAOD ~ ΔCOB by SAS Similarity
Hence, ∠A = ∠C and ∠B = ∠D. (Corresponding angles of similar triangles)

7. In ∆ABC and ∆DEF, ∠B = ∠E, ∠F = ∠C and AB = 3DE. Then, the two triangles are
(a) congruent but not similar (b) similar but not congruent
(c) neither congruent nor similar (d) congruent as well as similar
Ans: (b) similar but not congruent

2sin A  3cos A
8. If cosec A = 13/12, then the value of
4sin A  9cos A
(a) 4 (b) 5 (c) 6 (d) 3
Ans: (d) 3
Given cosec A = 13/12,
12 5
sin A  , cos A 
13 13
 12   5 
2    3 
2sin A  3cos A 13 13 24  15 9
Now,       3
4sin A  9cos A  12   5  48  45 3
4   9 
 13   13 

9. In the given figure, if TP and TQ are tangents to a circle with centre O, so that ∠POQ = 110°,
then ∠PTQ is

Prepared by: M. S. KumarSwamy, TGT(Maths) Page - 2 -


(a) 110° (b) 90° (c) 80° (d) 70°
Ans: (d) 70°

10. If the angle of elevation of the top of a tower from a point of observation at a distance of 100 m
from its base is 45°, then the height of the tower is:
(a) 160 m (b) 100 m (c) 200 m (d) 150 m
Ans: (b) 100 m
Here, PQ is the tower and A is a point of observation at a distance of 100 m from PQ.
PQ h
From right ΔAPQ,   tan 450  1  h  100m
AP 100

Thus, the height of tower is 100 metre.

11. The ratio in which x-axis divides the join of (2, -3) and (5, 6) is:
(a) 1: 2 (b) 3 : 4 (c) 1: 3 (d) 1: 5
Ans: (a) 1 : 2
Let P(x, 0) be the point on x-axis which divides the join of (2, -3) and (5, 6) in the ratio k : 1.
∴ By section formula,
 5k  2 6k  3 
P(x, 0) =  , 
 k 1 k 1 
6k  3 1
 y0  0  6k  3  0  k 
k 1 2
12. If tan θ = 1, then the value of sec θ + cosec θ is:
(a) 3√2 (b) 4√2 (c) 2√2 (d) √2
Ans: (c) 2√2
Given, tan θ = 1, we have θ = 45°
So, sec θ + cosec θ = √2 + √2 = 2√2.

13. If the area of circle is numerically equal to twice its circumference, then the diameter of the
circle is
(a) 4 units (b) 6 units (c) 8 units (d) 12 units
Ans: (c) 8 units

14. If the perimeter of a circle is equal to that of a square, then the ratio of the area of circle to the
area of the square is
(a) 14: 11 (b) 12: 13 (c) 11:14 (d) 13:12
Ans: (a) 14: 11
r
Here, it is given that 4s = 2πr  s 
2

Prepared by: M. S. KumarSwamy, TGT(Maths) Page - 3 -


Area of Circle  r 2  r 2 4 4  7 14
Now,  2  2 2   
Area of Square s  r  22 11
4
So, the ratio of the area of the circle to the area of square is 14 : 11.

15. The radii of 2 cylinders are in the ratio 2 : 3 and their heights are in the ratio 5 : 3. Then, the
ratio of their volumes is:
(a) 19 : 20 (b) 20 : 27 (c) 18:25 (d) 17:23
Ans: (b) 20 : 27
Let r1 and r2 be the two radii and h1 and h2 be the corresponding two heights of the two
cylinders. Then
r1 L h 5
 and 1  (Given)
r2 3 h2 3
V1  r12 h1 4 5 20
∴    
V2  r22 h2 9 3 27

16. For the following distribution:


Class 0-5 6-11 12-17 18-23 24-29
Frequency 13 10 15 8 11
the upper limit of the median class is
(a) 18.5 (b) 20.5 (c) 25.5 (d) 17.5
Ans:
Class Frequency Cf
– 0.5 – 5.5 13 13
5.5 – 11.5 10 23
11.5 – 17.5 15 38
17.5 – 23.5 8 46
23.5 – 29.5 11 57
n
Here, n = 57 So, = 28.5
2
The cumulative frequency, just greater than 28.5, is 38 which belongs to class 11.5 – 17.5.
So, the median class is 11.5 – 17.5 Its upper limit is 17.5

17. If the mean of the following distribution is 2.6, then the value of y is
Variable (x) 1 2 3 4 5
Frequency 4 5 y 1 2
(a) 3 (b) 8 (c) 13 (d) 24
Ans: (b) 8
Variable (x) 1 2 3 4 5 Total
Frequency (f) 4 5 y 1 2 y + 12
fx 4 10 3y 4 10 3y + 28
Here, ∑f = y + 12 and ∑fx = 3y + 28

Mean, x 
 f x  2.6  3 y  28  3 y  28  2.6 y  31.2
f y  12
 0.4 y  3.2  y  8

18. Two different dice are thrown together. The probability of getting the sum of the two numbers
less than 7 is:
(a) 5/12 (b) 7/12 (c) 12/5 (d) 3/11
Ans: (a) 5/12
Total outcomes = 36
Prepared by: M. S. KumarSwamy, TGT(Maths) Page - 4 -
Number of outcomes in which sum of two numbers is less than 7 = 15
∴ Required probability = 15/36 = 5/12

DIRECTION: In the question number 19 and 20, a statement of Assertion (A) is followed by a
statement of Reason (R).
Choose the correct option

19. Assertion: The HCF of two numbers is 9 and their LCM is 2016. If the one number is 54, then
the other number is 336.
Reason: Relation between numbers and their HCF and LCM is product of two numbers a, b =
HCF (a, b) × LCM (a, b).
(a) Both Assertion (A) and Reason (R) are true and Reason (R) is the correct explanation of
Assertion (A).
(b) Both Assertion (A) and Reason (R) are true but Reason (R) is not the correct explanation of
Assertion (A).
(c) Assertion (A) is true but Reason (R) is false.
(d) Assertion (A) is false but Reason (R) is true.
Ans: (a) Both assertion (A) and reason (R) are true and reason (R) is the correct explanation of
assertion (A)
Let the other number be x.
9 × 2016 = 54 × x
⇒ x = 336

20. Assertion (A): The value of y is 3, if the distance between the points P(2, -3) and Q(10, y) is 10.
Reason (R): Distance between two points is given by ( x2  x1 ) 2  ( y2  y1 )2
(a) Both Assertion (A) and Reason (R) are true and Reason (R) is the correct explanation of
Assertion (A).
(b) Both Assertion (A) and Reason (R) are true but Reason (R) is not the correct explanation of
Assertion (A).
(c) Assertion (A) is true but Reason (R) is false.
(d) Assertion (A) is false but Reason (R) is true.
Ans: (a) Both assertion (A) and reason (R) are true and reason (R) is the correct explanation of
assertion (A)
For y = 3
Distance PQ  (10  2)2  ( y  3)2  82  62  64  36  100  10 units

SECTION – B
Questions 21 to 25 carry 2 marks each.

21. For what value of k for which the following pair of linear equations have infinitely many
solutions: 2x + 3y = 7, (k – 1)x + (k + 2)y = 3k is
a b c
Ans: For a pair of linear equations to have infinitely many solutions, 1  1  1
a2 b2 c2
2 3 7 2 3
⇒    
k  1 k  2 3k k 1 k  2
Now, 2k + 4 = 3k – 3 ⇒ k = 7
and 9k = 7k + 14 ⇒ k = 7
Hence, the value of k is 7.

22. In the below left figure, two chords AB and CD intersect each other at the point P.

Prepared by: M. S. KumarSwamy, TGT(Maths) Page - 5 -


Prove that (i) ΔAPC ~ ΔDPB (ii) AP. PB = CP. DP
Ans: (i) Consider Δs APC and DPB
∠APC = ∠DPB (Vertically opopsite angles)
Also, ∠CAP = ∠PDB (Angles made by the same arc CB)
So, by AA similarity criteria,
ΔAPC ~ ΔDPB.
(ii) Corresponding sides of similar triangles are proportional
AP DP
∴ 
PC PB
⇒ AP.PB = CP.DP
OR
If in the given above right sided figure, AB || DE and BD || EF, then prove that DC2 = CF x AC

Ans: ΔABC in which DE || AB and BD || EF.


In ∆ABC, DE || AB
CD CE
⇒  ...(i) [Basic proportionality theorem]
AC BC
Again in ∆CDB, EF || BD
CF CE
⇒  ...(ii) [BPT]
CD BC
From (i) and (ii), we get
CD CF
  CD 2  AC  CF
AC CD

23. Two concentric circles are of radii 5 cm and 3 cm. Find the length of the chord of the larger
circle which touches the smaller circle.
Ans: Let O be the centre of the concentric circle of radii 5 cm and 3 cm respectively. Let AB be
a chord of the larger circle touching the smaller circle at P

Then AP = PB and OP⊥AB


Applying Pythagoras theorem in △OPA, we have
OA2 = OP2 + AP2 ⇒ 25 = 9 + AP2
⇒ AP2 = 16 ⇒ AP = 4 cm
∴ AB = 2AP = 8 cm

24. Evaluate: 3 cos2 60° sec2 30° – 2 sin2 30° tan2 60°.
Ans: 3 cos2 60° sec2 30° – 2 sin2 30° tan2 60°
Prepared by: M. S. KumarSwamy, TGT(Maths) Page - 6 -
2 2 2
1  2  1 2 3 4 1 3 1
= 3  
2  3
  2 
2
 3    2   3  1  
4 3 4 2 2

25. The length of the minute hand of a clock is 14 cm. Find the area swept by the minute hand in 5
minutes.
Ans: Area swept by the minute hand in 5 minutes will be the area of a sector of 30⁰ in a circle of
14 cm radius.

Area of sector of angle θ = πr²
3600
300 22 1 22 154
Area of sector of 30° = 0
 14 14   14  14 
360 7 12 7 3
= 51.33 cm²
Therefore, the area swept by the minute hand in 5 minutes is 51.33 cm²

OR
The radii of two circles are 8 cm and 6 cm respectively. Find the diameter of the circle having
area equal to the sum of the areas of the two circles.
Ans: Area of the circle = Area of first circle + Area of second circle
⇒ πR2 = π (r1)2 + π (r1)2
⇒ πR2 = π (8)2 + π (6)2 ⇒ πR2 = 64π + 36π
⇒ πR2 = 100π ⇒ R2 = 100 ⇒ R = 10. Thus, diameter of the circle = 2R = 20 cm.

SECTION – C
Questions 26 to 31 carry 3 marks each.
26. Prove that √5 is an irrational number.
p
Ans: Let 5 is a rational number then we have 5 , where p and q are co-primes.
q
 p  5q
Squaring both sides, we get p 2  5q 2
 p2 is divisible by 5  p is also divisible by 5
So, assume p = 5m where m is any integer.
Squaring both sides, we get p2 = 25m2
But p 2  5q 2
Therefore, 5q2 = 25m2  q2 = 5m2
 q2 is divisible by 5  q is also divisible by 5
From above we conclude that p and q have one common factor i.e. 5 which contradicts that p
and q are co-primes.
Therefore, our assumption is wrong.
Hence, 5 is an irrational number.

27. What number should be added to the polynomial x2 – 5x + 4 so that 3 is the zero of the
polynomial?
Ans: Let k be the number to be added to the given polynomial.
Then the polynomial becomes x2 – 5x + (4 + k)
As 3 is the zero of this polynomial, we get (3)2 – 5(3) + (4 + k) = 0
⇒ (4 + k) = 15 – 9 ⇒ 4 + k = 6 ⇒ k = 2
Thus, 2 is to be added to the given polynomial.

28. A part of monthly hostel charges in a college is fixed and the remaining depends on the number
of days one has taken food in the mess. When a student ‘A’ takes food for 22 days, he has to

Prepared by: M. S. KumarSwamy, TGT(Maths) Page - 7 -


pay Rs. 1380 as hostel charges; whereas a student ‘B’, who takes food for 28 days, pays Rs.
1680 as hostel charges. Find the fixed charges and the cost of food per day.
Ans: Let the fixed hostel charges be Rs. x and the cost of food per day be Rs. y.
According to the question, we get
x + 22y = 1380 ...(i)
and x + 28y = 1680 ...(ii)
Subtracting (i) from (ii), we get
6y = 300 ⇒ y = 300 ÷ 6 = 50
Putting y = 50 in (i), we get
x + 22(50) = 1380 ⇒ x + 1100 = 1380 ⇒ x = 280
∴ Fixed hostel charges = Rs. 280 and cost of the food per day = Rs. 50.
OR
Meena went to a bank to withdraw Rs 2,000. She asked the cashier to give her Rs. 50 and Rs.
100 notes only. Meena got 25 notes in all. How many notes of Rs. 50 and Rs. 100 she received?
Ans: Let Meena has received x no. of Rs. 50 notes and y no. of Rs. 100 notes.
So, 50x + 100y = 2000 ... (i)
x + y = 25 ... (ii)
Solving (i) and (ii), we get y = 15
Putting y = 15 in equation (ii), we get
x + 15 = 25
⇒ x = 10
Meena has received 10 pieces of Rs. 50 notes and 15 pieces of Rs. 100 notes.

29. In the given figure, OP is equal to diameter of the circle. Prove that ABP is an equilateral
triangle.

Ans: Join OP and let it meets the circle at point Q.


Since OP = 2r (Diameter of the circle)
Consider ΔAOP in which OA ⊥ AP and OP is the hypotenuse.

OA r 1
In ∆AOP, sin OPA     sin 300
OP 2r 2
 OPA  300
 APB  2OPA  600 (Centre lies on the bisector of the angle formed between two
tangents)
Now, PA = PB (Tangents from P)
⇒ ∠PAB = ∠PBA (∠s opposite to equal sides)
Using ∠APB and angle sum property of triangle, we have
∠PAB = ∠PBA = ∠APB
⇒∆APB is an equilateral triangle.
Prepared by: M. S. KumarSwamy, TGT(Maths) Page - 8 -
cos2  sin 2 
30. Prove that:   1  sin  cos 
1  tan  1  cot 
cos2  sin 2 
Ans: LHS  
1  tan  1  cot 
3
cos  sin 3 
 
cos   sin  cos   sin 
cos3   sin 3  (cos   sin  )(cos2   sin 2   cos  sin  )
 
cos   sin  cos   sin 
2 2
 cos   sin   cos sin 
 1  sin  cos   RHS
OR
If cos θ + sin θ = √2 cos θ, show that cos θ – sin θ = √2 sin θ.
Ans: Given, cos θ + sin θ = √2 cos θ
Squaring both sides, we get
(cos θ + sin θ)2 = (√2 cos θ)2
⇒ cos2θ + sin2θ + 2sin θ cos θ
⇒ 2sin θ cos θ = cos2θ – sin2θ
⇒ 2sin θ cos θ = (cos θ – sin θ)(cos θ + sin θ)
⇒ 2sin θ cos θ = (cos θ – sin θ) (√2 cos θ)
⇒ √2 sin θ = cos θ – sin θ ⇒ cos θ – sin θ = √2 sin θ

31. All the black face cards are removed from a pack of 52 playing cards. The reaming cards are
well shuffled and then a card is drawn at random. Find the probability of getting (i) face card (ii)
red card (iii) black card.
Ans: When all the black face cards are removed,
Remaining number of cards = 52 – 6 = 46
(i) Number of face cards in the remaining deck = 6
∴ P(getting a face card) = 6/46 = 3/23
(ii) Number of red cards in the remaining deck = 26
∴ P (getting a red card) = 26/46 = 13/23
(iii) Number of black cards in the remaining deck = 20
∴ P (getting a black card) = 20/46 = 10/23

SECTION – D
Questions 32 to 35 carry 5 marks each.
32. Some students planned a picnic. The total budget for food was Rs. 2,000. But 5 students failed
to attend the picnic and thus the cost of food for each member increased by Rs. 20. How many
students attended the picnic and how much did each student pay for the food?
Ans: Case I. Let number of students = x
and cost of food for each member = Rs. y
Then x × y = 2,000 ...(i)
Case II. New number of students = x – 5
New cost of food for each member = Rs. (y + 20)
Then (x – 5)(y + 20) = 2,000
⇒ xy + 20x – 5y – 100 = 2,000 ...(ii)
Solving (i) and (ii), we get x = –20, 25
x = –20 is rejected because number of students can’t be negative.
So, x = 25 ⇒ y = 80
Number of students = 25
Cost of food for each student = Rs. 80.
Prepared by: M. S. KumarSwamy, TGT(Maths) Page - 9 -
OR
If Zeba was younger by 5 years than what she really is, then the square of her age (in years)
would have been 11 more than five times her actual age. What is her age now?
Ans: Let the present age of Zeba be x years.
Age before 5 years = (x – 5) years
According to given condition, (x – 5)2 = 5x + 11
⇒ x2 + 25 – 10x = 5x + 11 ⇒ x2 – 10x – 5x + 25 – 11 = 0
⇒ x2 – 15x + 14 = 0 ⇒ x2 – 14x – x + 14 = 0
⇒ x (x – 14) – 1 (x – 14) = 0 ⇒ (x – 1) (x – 14) = 0
⇒ x – 1 = 0 or x – 14 = 0 ⇒ x = 1 or x = 14
But present age cannot be 1 year.
Hence, Present age of Zeba is 14 years.

33. Prove that “If a line is drawn parallel to one side of a triangle to intersect the other two sides in
distinct points, the other two sides are divided in the same ratio.”
In the figure, find EC if AD/DB = AE/EC using the above theorem.

Ans: For Theorem:


Figure, Given, To Prove and Construction - 2 marks
Proof - 2 marks
To find the value of EC = 9 cm – 1 mark

34. A tent is in the shape of a cylinder surmounted by a conical top. If the height and diameter of
the cylindrical part are 2.1 m and 4 m respectively, and the slant height of the top is 2.8 m, find
the area of the canvas used for making the tent. Also, find the cost of the canvas of the tent at
the rate of Rs. 500 per m2.
Ans: Radius (r) of the cylindrical part = Radius of the conical part = 4/2 m = 2 m

Area of the canvas used = CSA of the cylindrical part + CSA of the conical part
= 2πrh + πrl
22 22
= πr (2h + l)   2  (4.2  2.8)   2  7
7 7
2
= 44 m
Cost of the canvas = Rate per m2 × area of canvas
= Rs 500 × 44 = Rs. 22,000
OR

Prepared by: M. S. KumarSwamy, TGT(Maths) Page - 10 -


A rectangular metal block has length 15 cm, breadth 10 cm and height 5 cm. From this block, a
circular hole of diameter 7 cm is drilled out. Find: (i) the volume of the remaining solid (ii) the
surface area of the remaining solid.
Ans: (i) The volume of the remaining solid
= Volume of rectangular block – Volume of the circular hole

2
22  7 
= (15  10  5)      5 cm³
7 2
= 750 cm³ – 192.5 cm³ = 557.5 cm³
(ii) The surface area of the remaining solid
= Total surface area of the block – 2 (area of circle of the hole) + curved surface of circular hole
(cylinder)
= 2(l × b + b × h + h × l) – 2(πr2) + 2πrh
2
22  7  22 7
= 2(15  10  10  5  5 15)  2      5  2    5
7 2 7 2
= 550 cm² – 77 cm² + 110 cm² = 583 cm².
35. The distribution below gives the makes of 100 students of a class, if the median makes are 24,
find the frequencies f1 and f2
Marks 0-5 5-10 10-15 15-20 20-25 25-30 30-35 35-40
No. of students 4 6 10 f1 25 f2 18 5
Ans:
Class Frequency cf
0–5 4 4
5 – 10 6 10
10 – 15 10 20
15 – 20 f1 20 + f1
20 – 25 25 45 + f1
25 – 30 f2 45 + f1 + f2
30 – 35 18 63 + f1 + f2
35 – 40 5 68 + f1 + f2
Now, Median = 24 (Given)
So, median class = 20 – 25
For this class,
I = 20. h = 5, N2 = 50, cf = 20 + f1, f= 25
n 
 2  cf 
We know, Median = l    h
 f 
 
50  (20  f1 ) 30  f1
 24  20  5  4   30  f1  20  f1  10
25 5
Also, sum of frequencies = 100
⇒ 68 + f1 + f2 = 100 ⇒ f1 + f2 = 32 ⇒ 10 + f2 = 32 ⇒ f2 = 22
∴ f1 = 10, f2 = 22.
Prepared by: M. S. KumarSwamy, TGT(Maths) Page - 11 -
SECTION – E(Case Study Based Questions)
Questions 36 to 38 carry 4 marks each.

36. Case Study – 1


Ram is watching the top and bottom of a lighthouse from the top of the building. The angles of
elevation and depression of the top and bottom of a lighthouse from the top of a 60 m high
building are 30° and 60° respectively.

Find (i) the difference between the heights of the lighthouse and the building.
(ii) the distance between the lighthouse and the building.
OR
The ratio of the height of a light house and the length of its shadow on the ground is √3 : 1 What
is the angle of elevation?
Ans: In right ∆ABD,
AB 60 60
tan 600   3  BD   20 3m
BD BD 3
∴ AE = 20√3 m (∵ BD = AE)
Now in right ∆AEC
CE 1 CE
tan 300     CE  20m
AE 3 20 3
(i) Difference between the heights of the lighthouse and the building = CE = 20 m
(ii) The distance between the lighthouse and the building = BD = 20√3 m.
OR
Let AB be the light house, BC be its shadow and θ be the angle of elevation of the sun at that
instant

AB
Then, in triangle ABC, we have, tan θ =
BC
3
tan θ =  tan 600 ⇒ θ = 60°
1
Hence, angle of elevation of the sun is 60°.

Prepared by: M. S. KumarSwamy, TGT(Maths) Page - 12 -


37. Case Study – 2
Saving money is a good habit and it should be inculcated in children from the beginning. A
father brought a piggy bank for his son Aditya. He puts one five-rupee coin of his savings in the
piggy bank on the first day. He increases his savings by one five-rupee coin daily.

(i) If the piggy bank can hold 190 coins of five rupees in all, find the number of days he can
contribute to put the five-rupee coins into it
(ii) Find the total money he saved.
OR
If 6 times the 6th term of an A.P., is equal to 9 times the 9th term, find its 15th term.
Ans: Child's savings day wise are 5, 10, 15, 20, 25, ....... to n days
We can have at most 190 coins
i.e., 1 + 2 + 3 + 4 + 5 + .... to n term = 190
n
⇒ [2  1  ( n  1)1]  190
2
⇒ n(n + 1) = 380 ⇒ n2 + n - 380 = 0
⇒ (n + 20)(n−19) = 0 ⇒ (n + 20)(n - 19) = 0
⇒ n = −20 or n = 19
But number of coins cannot be negative
n = 19 (rejecting n = -20)
So, number of days = 19
Total money she saved = 5 + 10 + 15 + 20 + ...
= 5 + 10 + 15 + 20 + ... upto 19 terms
19 19 19
= [2  5  (19  1)5]  [10  90]  [100]  19  50  950
2 2 2
OR
Let, the first term of A.P. be ‘a’ and its common difference be ‘d’
Given, 6a6 = 9a9
⇒ 6(a + 5d) = 9(a + 8d)
⇒ 6a + 30d = 9a + 72d
⇒ -3a = 42d ⇒ a = -14d …(i)
Then, 15th term i.e. a15 = a + 14d = -14d + 14d [from (i)]
=0
Hence, 15th term of A.P. is 0.

38. Case Study – 3


In a GPS, The lines that run east-west are known as lines of latitude, and the lines running
north-south are known as lines of longitude. The latitude and the longitude of a place are its
coordinates and the distance formula is used to find the distance between two places. The
distance between two parallel lines is approximately 150 km. A family from Uttar Pradesh
planned a round trip from Lucknow (L) to Puri (P) via Bhuj (B) and Nashik (N) as shown in the
given figure below.

Prepared by: M. S. KumarSwamy, TGT(Maths) Page - 13 -


Based on the above information answer the following questions using the coordinate geometry.
(i) Find the distance between Lucknow (L) to Bhuj(B).
(ii) If Kota (K), internally divide the line segment joining Lucknow (L) to Bhuj (B) into 3 : 2
then find the coordinate of Kota (K).
(iii) Name the type of triangle formed by the places Lucknow (L), Nashik (N) and Puri (P)
OR
Find a place (point) on the longitude (y-axis) which is equidistant from the points Lucknow (L)
and Puri (P).
(i)

(ii)

(iii)

OR

Prepared by: M. S. KumarSwamy, TGT(Maths) Page - 14 -


Prepared by: M. S. KumarSwamy, TGT(Maths) Page - 15 -
KENDRIYA VIDYALAYA GACHIBOWLI, GPRA CAMPUS, HYD-32
SAMPLE PAPER TEST 03 FOR BOARD EXAM 2023
(ANSWERS)
SUBJECT: MATHEMATICS MAX. MARKS : 80
CLASS : X DURATION : 3 HRS
General Instruction:
1. This Question Paper has 5 Sections A-E.
2. Section A has 20 MCQs carrying 1 mark each.
3. Section B has 5 questions carrying 02 marks each.
4. Section C has 6 questions carrying 03 marks each.
5. Section D has 4 questions carrying 05 marks each.
6. Section E has 3 case based integrated units of assessment (04 marks each) with sub-parts of the
values of 1, 1 and 2 marks each respectively.
7. All Questions are compulsory. However, an internal choice in 2 Qs of 5 marks, 2 Qs of 3 marks
and 2 Questions of 2 marks has been provided. An internal choice has been provided in the
2marks questions of Section E
8. Draw neat figures wherever required. Take π =22/7 wherever required if not stated.
SECTION – A
Questions 1 to 20 carry 1 mark each.
1. The vertices of a parallelogram in order are A(1, 2), B(4, y), C(x, 6) and D(3, 5). Then (x, y) is:
(a) (6, 3) (b) (3, 6) (c) (5, 6) (d) (1, 4)
Ans: (a) (6, 3)

2. The ratio of LCM and HCF of the least composite and the least prime numbers is:
(a) 1: 2 (b) 2: 1 (c) 1: 1 (d) 1: 3 1
Ans. (b) 2: 1
Least composite number is 4 and the least prime number is 2
LCM(4, 2): HCF (4, 2) = 4: 2 = 2: 1

3. The value of k for which the lines 5x + 7y = 3 and 15x + 21y = k coincide is:
(a) 9 (b) 5 (c) 7 (d) 18
Ans: (a) 9
a b c
For coincident lines we have 1  1  1
a2 b2 c2
5 7 3 1 3
     k 9
15 21  k 3 k
1 1
4. Write a quadratic polynomial whose sum of zeroes is and product of zeroes is .
4 4
2 2
(a) 4x + x + 1 (b) x + 4x – 1
(c) 2x2 + 3x – 1 (d) x2 – 2x + 1

Prepared by: M. S. KumarSwamy, TGT(Maths) Page - 1 -


Ans: (a) 4x2 + x + 1
1 1
Sum of zeroes = and Product of zeroes =
4 4
∴ Quadratic Polynomial is p(x) = x2 – (sum of zeroes) x + product of zeroes
 1  1 1
∴ p(x) = x2 –   x + = 0 ⇒ p(x) = (4x2 + x + 1)
 4 4 4
Hence, required quadratic polynomial is 4x2 + x + 1

5. ΔABC~ΔPQR. If AM and PN are altitudes of ΔABC and ΔPQR respectively and AB2: PQ2 = 4:
9, then AM: PN =
(a) 16: 81 (b) 4: 9 (c) 3: 2 (d) 2: 3
Ans: (d) 2 : 3

6. In ΔABC right angled at B, if tanA = √3 , then then cosA cosC – sinAsinC =


(a) –1 (b) 0 (c) 1 (d) √3 / 2
Ans: (b) 0
tan A = √3 = tan 60°, so, ∠A = 60°,
Hence, ∠C = 30°.
1 3 3 1
So, cos A cos C – sin A sin C     0
2 2 2 2

7. If 2sin2 β – cos2 β = 2, then β is:


(a) 0° (b) 90° (c) 45° (d) 30° 1
Ans. (b) 90°
2 sin2 β – cos2 β = 2
Then, 2 sin2 β – (1 – sin2 β) = 2
⇒ 3sin2 β = 3 or sin2 β = 1 ⇒ β is 90°.

8. If LCM(x, 18) = 36 and HCF(x, 18) = 2, then x is:


(a) 2 (b) 3 (c) 4 (d) 5 1
Ans. (c) 4
LCM × HCF = Product of two numbers
⇒ 36 × 2 = 18 × x
⇒x=4

Prepared by: M. S. KumarSwamy, TGT(Maths) Page - 2 -


9. ABCD is a trapezium with AD ∥ BC and AD = 4cm. If the diagonals AC and BD intersect each
other at O such that AO/OC = DO/OB =1/2, then BC =
(a) 6cm (b) 7cm (c) 8cm (d) 9cm
Ans: (c) 8cm

10. In the figure, if DE || BC, AD = 3 cm, BD = 4 cm and BC = 14 cm, then DE equals :


(a) 7 cm (b) 6 cm (c) 4 cm (d) 3 cm

Ans: (b) 6 cm
∵ DE || BC
∴ ∠ADE = ∠ABC [corresonding angles] ....(i)
Now, in ΔADE and ΔABC,
∠ADE = ∠ABC [Proved in (i)]
∠A = ∠A [Common angle]
∴ ΔADE ~ ΔABC [By AA similarity axiom]
AD DE
  [∵ Corresponding sides of similar triangles are proportional]
AB BC
AD DE 3 DE 3 DE
       DE  6
AD  BD BC 3  4 14 7 14

11. In the given figure, PA and PB are tangents to the circle with centre O. If ∠APB = 60°, then
∠OAB is

(a) 30° (b) 60° (c) 90° (d) 15°


Ans: (a) 30°
PA = PB ( Tangents drawn from external point are equal)
⇒ ∠ABP = ∠BAP = x ( Angles opposite to equal sides are equal)
In ∆APB, 60° + x + x = 180°
⇒ 2x = 120° ⇒ x = 60°
Now, ∠OAP = 90° (∵ Tangent is perpendicular to the radius through the point of contact)
∴ ∠OAB = 90° – 60° = 30°

Prepared by: M. S. KumarSwamy, TGT(Maths) Page - 3 -


12. If the difference of Mode and Median of a data is 24, then the difference of median and mean is
(a) 8 (b) 12 (c) 24 (d) 36
Ans: (b) 12

13. A horse is tied to a pole with 28 m long rope. The perimeter of the field where the horse can
graze is (Take π = 22/7)
(a) 60 cm (b) 85 cm (c) 124 cm (d) 176 cm
Ans: (d) 176 cm
Horse can graze in the field which is a circle of radius 28 cm.
So, required perimeter = 2πr = 2π (28) cm = 2 × 22/7 × 28 cm = 176 cm

14. Two dice are thrown at the same time. The probability of getting not doublet is
(a) 1/3 (b) 1/6 (c) 1/5 (d) 5/6
Ans: (d) 5/6
Total number of possible outcomes = 36
Number of outcomes with doublets = 6 i.e. (1, 1), (2, 2), (3, 3), (4, 4), (5, 5), (6, 6)
So, number of favourable outcomes = 36 – 6 = 30
∴ Required probability = 30/36 = 5/6

15. For the following distribution:


Class 0-5 5-10 10-15 15-20 20-25
Frequency 10 15 12 20 9
the sum of lower limits of the median class and modal class is
(a) 15 (b) 25 (c) 30 (d) 35
Ans: (b) 25

5sin   3cos 
16. If 5 tan θ = 4, then the value of is
5sin   2cos 
(a) 1/6 (b) 1/7 (c) 1/4 (d) 1/5
Ans: (a) 1/6

5sin   3cos  5 tan   3


 [Dividing numerator and denominator by cos θ]
5sin   2cos  5 tan   2

Prepared by: M. S. KumarSwamy, TGT(Maths) Page - 4 -


4
5  3
5 43 1
  
4
5  2 4  2 6
5

17. The ratio of the volumes of two spheres is 8 : 27. The ratio between their surface areas is
(a) 2 : 3 (b) 4 : 27 (c) 8 : 9 (d) 4 : 9
Ans: (d) 4 : 9

18. The area of the circle that can be inscribed in a square of 6cm is
(a) 36π cm2 (b) 18π cm2 (c) 12 π cm2 (d) 9π cm2
2
Ans: (d) 9π cm

DIRECTION: In the question number 19 and 20, a statement of Assertion (A) is followed by a
statement of Reason (R).
Choose the correct option

19. Assertion (A): The point (–1, 6) divides the line segment joining the points (–3, 10) and (6, –8)
in the ratio 2 : 7 internally.
Reason (R): Given three points, i.e. A, B, C form an equilateral triangle, then AB = BC = AC.
(a) Both assertion (A) and reason (R) are true and reason (R) is the correct explanation of
assertion (A)
(b) Both assertion (A) and reason (R) are true and reason (R) is not the correct explanation of
assertion (A)
(c) Assertion (A) is true but reason (R) is false.
(d) Assertion (A) is false but reason (R) is true.
Ans: (b) Both assertion (A) and reason (R) are true and reason (R) is not the correct
explanation of assertion (A)

20. Assertion (A): The number 6n, n being a natural number, ends with the digit 5.
Reason (R): The number 9n cannot end with digit 0 for any natural number n.
(a) Both assertion (A) and reason (R) are true and reason (R) is the correct explanation of
assertion (A)
(b) Both assertion (A) and reason (R) are true and reason (R) is not the correct explanation of
assertion (A)
(c) Assertion (A) is true but reason (R) is false.
(d) Assertion (A) is false but reason (R) is true.
Ans: (d) Assertion (A) is false but reason (R) is true.

Prepared by: M. S. KumarSwamy, TGT(Maths) Page - 5 -


SECTION – B
Questions 21 to 25 carry 2 marks each.

21. If 217x + 131y = 913, 131x + 217y = 827, then find the value of x and y
Ans: Adding the two equations and dividing by 348, we get : x + y = 5
Subtracting the two equations and dividing by 86, we get : x – y = 1
Solving these two new equations, we get, x = 3 and y = 2

22. If sin(A + B) = 1 and cos(A – B) = √3/2, 0°< A + B ≤ 90° and A > B, then find the measures of
angles A and B.
Ans: sin(A + B) = 1 = sin 900, so A + B = 900 ……….(i)
cos(A – B) = √3/2 = cos 300, so A – B = 300 ………(ii)
From (i) & (ii) ∠A = 60° and ∠B = 30°
OR
cos   sin  1  3
Find an acute angle θ when 
cos   sin  1  3
cos   sin  1  3
Ans: 
cos   sin  1  3

23. In the given figure below, AD/AE=AC/BD and ∠1=∠2. Show that Δ BAE~ ΔCAD .

Ans: In ΔABC, ∠1 = ∠2
∴ AB = BD ………(i)
AD AC
Given, 
AE BD
AD AC
Using equation (i), we get  ……….(ii)
AE AB
AC AD
In ΔBAE and ΔCAD, by equation (ii), 
AB AE
and ∠A= ∠A (common)
∴ ΔBAE ~ ΔCAD [By SAS similarity criterion]

24. The length of the minute hand of a clock is 6cm. Find the area swept by it when it moves from
5:25 pm to 6:00 pm.
Ans: We know that, in 60 minutes, the tip of minute hand moves 360°
In 1 minute, it will move = 360°/60 = 6°
∴ From 5:25 pm to 6:00 pm i.e. 35 min, it will move through = 35 × 6° = 210°

Prepared by: M. S. KumarSwamy, TGT(Maths) Page - 6 -


∴ Area of swept by the minute hand in 35 min = Area of sector with sectorial angle θ of 210°
and radius of 6 cm

OR
In the given figure, the shape of the top of a table is that of a sector of a circle with centre O and
∠AOB = 90°. If AO = OB = 42 cm, then find the perimeter of the top of the table is [Take π =
22/7]

Ans: Perimeter = length of major arc + 2r

25. A circle is inscribed in a ΔABC having AB= 10cm, BC = 12cm and CA = 8cm and touching
these sides at D, E, F respectively. Find the lengths of AD, BE and CF

Ans: Let AD = AF = x cm,


BD = BE = y cm
and CE = CF = z cm

Now, x + y = AB = 10 cm
y + z = BC = 12 cm
z + x = CA = 8 cm
Adding all we get 2(x+ y +z) = 30
Prepared by: M. S. KumarSwamy, TGT(Maths) Page - 7 -
⇒ x + y + z = 15
Subtracting, we get z = 5 cm, x = 3 cm and y = 7 cm
Hence, AD = 3 cm, BE = 7 cm and CF = 5 cm.

SECTION – C
Questions 26 to 31 carry 3 marks each.
26. Given that √5 is irrational, prove that 2 + 3√5 is irrational.
Ans: Let us assume 2 + 3√5 is rational, then it must be in the form of p/q where p and q are co-
prime integers and q ≠0
p
i.e. 2 + 3√5 =
q
p  2q
So 5  …(i)
3q
Since p, q, 5 and 2 are integers and q ≠ 0,
RHS of equation (i) is rational.
But LHS of (i) is √5 which is irrational. This is not possible.
This contradiction has arisen due to our wrong assumption that 2 + 3√5 is rational.
So, 2 + 3√5 is irrational.

27. Find the zeroes of the polynomial x2 + x – 2, and verify the relation between the coefficients
and the zeroes of the polynomial.
1
Ans: Now we have given the polynomial: x2 + x–2=0
6
Simplifying it, we get 6x2 + x – 12 = 0
⇒ 6x2 – 8x + 9x – 12 = 0
⇒ (6x2 – 8x) + (9x – 12) = 0
⇒ 2x(3x – 4) + 3(3x – 4) = 0
⇒ (3x – 4)(2x + 3) = 0
⇒ x = 4/3 or x = -3/2
Here, a = 6, b = 1, c = –12
4  3  8  9 1 b
Sum of zeroes =       
3  2 6 6 a
4  3  12 c
Product of zeroes =      
3  2 6 a

28. A train covered a certain distance at a uniform speed. If the train would have been 6 km/h faster,
it would have taken 4 hours less than the scheduled time. And, if the train were slower by 6
km/hr; it would have taken 6 hours more than the scheduled time. Find the length of the
journey.
Ans: Let the actual speed of the train be x km/hr and let the actual time taken be y hours.
Distance covered is xy km If the speed is increased by 6 km/hr, then time of journey is reduced
by 4 hours i.e., when speed is (x+6)km/hr, time of journey is (y−4) hours.
∴ Distance covered = (x + 6)(y − 4)
⇒ xy = (x + 6)(y − 4) ⇒ −4x + 6y − 24 = 0 ⇒ −2x + 3y −12 = 0 ………….(i)
Similarly xy = (x − 6)(y + 6) ⇒ 6x − 6y − 36 = 0 ⇒ x − y − 6 = 0 …………(ii)
Solving (i) and (ii) we get x=30 and y=24
Putting the values of x and y in equation (i), we obtain
Distance = (30 × 24)km =720km.
Hence, the length of the journey is 720km.
OR

Prepared by: M. S. KumarSwamy, TGT(Maths) Page - 8 -


Anuj had some chocolates, and he divided them into two lots A and B. He sold the first lot at the
rate of ₹2 for 3 chocolates and the second lot at the rate of ₹1 per chocolate, and got a total of
₹400. If he had sold the first lot at the rate of ₹1 per chocolate, and the second lot at the rate of
₹4 for 5 chocolates, his total collection would have been ₹460. Find the total number of
chocolates he had.
Ans: Let the number of chocolates in lot A be x
And let the number of chocolates in lot B be y
∴ total number of chocolates =x+y
2
Price of 1 chocolate = ₹ 2/3 , so for x chocolates = x
3
and price of y chocolates at the rate of ₹ 1 per chocolate =y.
2
∴ by the given condition x + y = 400 ⇒ 2x + 3y = 1200 ..............(i)
3
4
Similarly, x + y = 460 ⇒ 5x + 4y = 2300 ........ (ii)
5
Solving (i) and (ii) we get x = 300 and y = 200
∴ x + y = 300 + 200 = 500
So, Anuj had 500 chocolates.

sin   cos   1
29. Prove that:  sec   tan 
sin   cos   1
tan   1  sec 
Ans: LHS = tan   1  sec  (Dividing numerator and denominator by cos)

tan   sec   1

tan   1  sec 
tan   sec   (sec 2   tan 2  )

tan   1  sec 
(sec   tan  )(1  sec   tan  )

tan   1  sec 
 sec   tan  = RHS

30. Prove that a parallelogram circumscribing a circle is a rhombus


Ans: We have ABCD, a parallelogram which circumscribes a circle (i.e., its sides touch the
circle) with centre O.
Since tangents to a circle from an external point are equal in length,
∴ AP = AS, BP = BQ, CR = CQ and DR = DS
Adding, we get
(AP + BP) + (CR + DR) = (AS + DS) + (BQ + CQ)
⇒ AB + CD = AD + BC
But AB = CD [opposite sides of ABCD]
and BC = AD
∴ AB + CD = AD + BC ⇒ 2 AB = 2 BC
⇒ AB = BC
Similarly AB = DA and DA = CD
Thus, AB = BC = CD = AD
Hence ABCD is a rhombus.

OR

Prepared by: M. S. KumarSwamy, TGT(Maths) Page - 9 -


In the figure XY and X'Y' are two parallel tangents to a circle with centre O and another tangent
AB with point of contact C interesting XY at A and X'Y' at B, what is the measure of ∠AOB.

Ans: Join OC. Since, the tangents drawn to a circle from an external point are equal.
∴ AP = AC

In Δ PAO and Δ AOC, we have:


AO = AO [Common]
OP = OC [Radii of the same circle]
AP = AC
⇒ Δ PAO ≅ Δ AOC [SSS Congruency]
∴ ∠PAO = ∠CAO = ∠1
∠PAC = 2 ∠1 ...(1)
Similarly ∠CBQ = 2 ∠2 ...(2)
Again, we know that sum of internal angles on the same side of a transversal is 180°.
∴ ∠PAC + ∠CBQ = 180°
⇒ 2 ∠1 + 2 ∠2 = 180° [From (1) and (2)]
⇒ ∠1 + ∠2 = 180°/2 = 90° ...(3)
Also ∠1 + ∠2 + ∠AOB = 180° [Sum of angles of a triangle]
⇒ 90° + ∠AOB = 180°
⇒ ∠AOB = 180° − 90°
⇒ ∠AOB = 90°.

31. Two coins are tossed simultaneously. What is the probability of getting
(i) At least one head? (ii) At most one tail? (iii) A head and a tail?
Ans: Total number of outcomes = 4
(i) Number of outcomes with at least one head = 3
∴ Required probability = 3/4
(ii) Number of outcomes with at most one tail = 3
∴ Required probability = 3/4
(iii)Number of outcomes with a head and a tail = 2
∴ Required probability = 2/4 = 1/2

Prepared by: M. S. KumarSwamy, TGT(Maths) Page - 10 -


SECTION – D
Questions 32 to 35 carry 5 marks each.
1
32. Two pipes running together can fill a cistern in 3 hours. If one pipe takes 3 hours more than
13
the other to fill it, find the time in which each pipe would fill the cistern.
Ans: Let time taken by faster pipe to fill the cistern be x hrs.
Therefore, time taken by slower pipe to fill the cistern = (x + 3) hrs
Since the faster pipe takes x minutes to fill the cistern.
1
∴ Portion of the cistern filled by the faster pipe in one hour =
x
1
Portion of the cistern filled by the slower pipe in one hour =
x3
13
Portion of the cistern filled by the two pipes together in one hour =
40
1 1 13 x  3  x 13
According to the question,    
x x  3 40 x( x  3) 40
⇒ 40 (2x + 3) = 13x (x + 3) ⇒ 80x + 120 = 13x2 + 39x
⇒ 13x2 – 41x – 120 = 0 ⇒ 13x2 – 65x + 24x – 120 = 0
⇒ 13x (x – 5) + 24 (x – 5) = 0 ⇒ (x – 5) (13x + 24) = 0
⇒ Either x – 5 = 0 or 13x + 24 = 0
⇒ x = 5 as x = −24/13 not possible.
Hence, the time taken by the two pipes is 5 hours and 8 hours respectively.
OR
In a flight of 600km, an aircraft was slowed down due to bad weather. Its average speed for the
trip was reduced by 200 km/hr from its usual speed and the time of the flight increased by 30
min. Find the scheduled duration of the flight.
Ans: Let the usual speed of plane be x km/hr
and the reduced speed of the plane be (x – 200) km/hr
Distance =600 km [Given]
According to the question,
(time taken at reduced speed) – (Schedule time) = 30 minutes = 0.5 hours.
600 600 1
⇒  
x  200 x 2
Which on simplification gives: x2 – 200x − 240000 = 0
⇒ x2 – 600x + 400x −240000 = 0
⇒ x(x – 600) + 400( x – 600) = 0 ⇒ (x – 600)(x + 400) =0 ⇒ x = 600 or x = −400
But speed cannot be negative.
∴ The usual speed is 600 km/hr and
the scheduled duration of the flight is 600/600 =1hour

33. Prove that if a line is drawn parallel to one side of a triangle intersecting the other two sides in
distinct points, then the other two sides are divided in the same ratio.
In the figure, find EC if AD/DB = AE/EC using the above theorem.

Ans: For the Theorem :


Given, To prove, Construction and figure of 2 marks
Prepared by: M. S. KumarSwamy, TGT(Maths) Page - 11 -
Proof of 2 marks
AD AE 2 3 1 3
Using Thales theorem, we get       EC  9cm 1 mark
DB EC 6 EC 3 EC

34. A cubical block of side 10 cm is surmounted by a hemisphere. What is the largest diameter that
the hemisphere can have? Find the cost of painting the total surface area of the solid so formed,
at the rate of Rs. 5 per 100 sq. cm. [Use π = 3.14]
Ans: Side of the cubical block (l) = 10 cm.

The hemisphere is surmounted on it.


The largest diameter the hemisphere can have =
side of the cubical block
Diameter of the hemisphere = 10 cm
Radius of the hemisphere (r) = 5 cm
Total surface area of the solid formed
= TSA of the cubical + CSA of the hemisphere - Area of the base of the hemisphere
= 6l2 + 2πr2 – πr2 = 6l2 + πr2
= 6 × (10)2 + 3.14 × (5)2
= 6 × 100 + 3.14 × 25
= 600 + 78.50
= 678.5 cm2
Rate of painting = Rs. 5 per 100 cm2
Cost pf painting the solid formed = Rs. 5/100 × 678.5
= Rs. 33.925 = Rs. 33.93 (approx)

OR
Due to heavy floods in a state, thousands were rendered homeless. 50 schools collectively
decided to provide place and the canvas for 1500 tents and share the whole expenditure equally.
The lower part of each tent is cylindrical with base radius 2.8 m and height 3.5 m and the upper
part is conical with the same base radius, but of height 2.1 m. If the canvas used to make the
tents costs ₹120 per m2, find the amount shared by each school to set up the tents.
Ans: Radius of the base of cylinder (r) = 2.8 m = Radius of the base of the cone (r)
Height of the cylinder (h)=3.5 m
Height of the cone (H)=2.1 m.
Slant height of conical part (l)=√(r2 + H2) = √[(2.8)2 + (2.1)2] = √(7.84 + 4.41) = √12.25 = 3.5 m
Area of canvas used to make tent = CSA of cylinder + CSA of cone = 2πrh + πrl
22
= πr(2h + l) =  2.8  (7  3.5)  22  0.4  10.5  92.4m2
7
Cost of 1500 tents at ₹120 per sq.m = 1500 × 120 × 92.4 = 1,66,32,000
Share of each school to set up the tents = 16632000/50 = ₹3,32,640

Prepared by: M. S. KumarSwamy, TGT(Maths) Page - 12 -


35. The median of the following data is 868. Find the values of x and y, if the total frequency is 100
Class Frequency
800 – 820 7
820 – 840 14
840 – 860 x
860 – 880 25
880 – 900 y
900 – 920 10
920 – 940 5
Ans:
Class Frequency Frequency
800 – 820 7 7
820 – 840 14 21
840 – 860 x x + 21
860 – 880 25 x + 46
880 – 900 y x + y + 46
900 – 920 10 x + y + 56
920 – 940 5 x + y + 61

From table, we have x + y + 61 = 100 ⇒ x + y = 100 – 61 ⇒ x + y = 39


Here, median = 868, therefore median class is 860 – 880
So, l = 860, cf = x + 21, f = 25, h = 20, n/2 = 50
n 
 2  cf   50  ( x  21) 
Now, Median  l    h   868  860    20 
 f   25 
 
 50  x  21)  29  x
 868  860    4  8  4
 5  5
 40  (29  x )4  29  x  10  x  29  10  19
 y  39  19  20

SECTION – E(Case Study Based Questions)


Questions 36 to 38 carry 4 marks each.

36. Case Study-1


The school auditorium was to be constructed to accommodate at least 1500 people. The chairs
are to be placed in concentric circular arrangement in such a way that each succeeding circular
row has 10 seats more than the previous one.

(i) If the first circular row has 30 seats, how many seats will be there in the 10th row? (1)
(ii) For 1500 seats in the auditorium, how many rows need to be there? (2)

Prepared by: M. S. KumarSwamy, TGT(Maths) Page - 13 -


OR
If 1500 seats are to be arranged in the auditorium, how many seats are still left to be put after
10th row? (2)
(iii) If there were 17 rows in the auditorium, how many seats will be there in the middle row?(1)
Ans: (i) Since each row is increasing by 10 seats, so it is an AP with first term a = 30, and
common difference d =10. So number of seats in 10th row = 10 = a + 9d
= 30 + 9 ×10 = 120
n n
(ii) Sn= ( 2a + (n – 1)d) ⇒ 1500 = ( 2 × 30 + (n-1)10)
2 2
⇒ 3000 = 50n + 10n2 ⇒ n2 + 5n – 300 = 0
⇒ n2 + 20n – 15n – 300 = 0 ⇒ (n + 20) (n – 15) =0
Rejecting the negative value, n = 15
OR
No. of seats already put up to the 10th row = S10
10
S10 = {2 × 30 + (10-1)10)} = 5(60 + 90) = 750
2
So, the number of seats still required to be put are 1500 – 750 = 750
(iii) If no. of rows =17
then the middle row is the 9th row
9= a + 8d = 30 + 80 = 110 seats

37. Case Study-2


The diagrams show the plans for a sun room. It will be built onto the wall of a house. The four
walls of the sunroom are square clear glass panels. The roof is made using
• Four clear glass panels, trapezium in shape, all the same size
• One tinted glass panel, half a regular octagon in shape

( i) Find the mid-point of the segment joining the points J (6, 17) and I (9, 16).
(1)
(ii) Find the distance between the points A and S. (1)
(iii) Find the co-ordinates of the point which divides the line segment joining the points A and B
in the ratio 1:3 internally. (2)
OR
Prepared by: M. S. KumarSwamy, TGT(Maths) Page - 14 -
(iii) If a point (x,y) is equidistant from the Q(9,8) and S(17,8),then find the relation between x
and y. (2)
 6  9 17  16   15 33 
Ans: (i) Mid-point of JI =  ,  , 
 2 2   2 2
(ii) Distance between A and S = 16 boxes.
(iii) Coordinates of A and B are (1, 8) and (5, 10) respectively.
Coordinates of point dividing AB in the ratio 1 : 3 internally are:
1  5  3 1 1 10  3  8 8 34
x ,y  x   2, y   8.5
1 3 1 3 4 4
Co-ordinates of required points be (2, 8.5)
OR
(iii) Let P (x,y) is equidistant from the Q(9,8) and S(17,8) then we have
PQ = PS ⇒ PQ2 = PS2
⇒ (x – 9)2 + (y – 8)2 = (x – 17)2 + (y – 8)2
⇒ (x – 9)2 = (x – 17)2
⇒ x2 – 18x + 81 = x2 – 34x + 289
⇒ 34x – 18x + 81 – 289 = 0
⇒ 16x – 208 = 0
⇒ x – 13 = 0

38. Case Study – 3


Anita purchased a new building for her business. Being in the prime location, she decided to
make some more money by putting up an advertisement sign for a rental ad income on the roof
of the building.

From a point P on the ground level, the angle of elevation of the roof of the building is 30° and
the angle of elevation of the top of the sign board is 45°. The point P is at a distance of 24 m
from the base of the building.

Prepared by: M. S. KumarSwamy, TGT(Maths) Page - 15 -


On the basis of the above information, answer the following questions:
(i) Find the height of the building (without the sign board). (2)
OR
Find the height of the building (with the sign board) (2)
(ii) Find the height of the sign board. (1)
(iii) Find the distance of the point P from the top of the sign board. (1)
Ans: (i) In ∆APC,

tan 30° = AB/AP


⇒ 1/√3 = AB/24
⇒ AB = 24/√3 m = 13.85 m = 14 m (approx)
OR
Considering, the diagram in the above question, AC as the new height of the shop including the
sign-baard.
In ∆APC,
tan 45° = AC/AP
⇒ 1 = AC/24
⇒ AC = 24 m

(ii) From Q (i) and Q (ii).


Length of sign board, BC = AC – AB
= 24 – 14
= 10 m

(iii) In ∆APC,
cos 45° = AP/AC
⇒ 1/√2 = 24/AC
⇒ PC = 24√2 m

Prepared by: M. S. KumarSwamy, TGT(Maths) Page - 16 -


KENDRIYA VIDYALAYA GACHIBOWLI, GPRA CAMPUS, HYD-32
SAMPLE PAPER TEST 04 FOR BOARD EXAM 2023
(ANSWERS)
SUBJECT: MATHEMATICS MAX. MARKS : 80
CLASS : X DURATION : 3 HRS
General Instruction:
1. This Question Paper has 5 Sections A-E.
2. Section A has 20 MCQs carrying 1 mark each.
3. Section B has 5 questions carrying 02 marks each.
4. Section C has 6 questions carrying 03 marks each.
5. Section D has 4 questions carrying 05 marks each.
6. Section E has 3 case based integrated units of assessment (04 marks each) with sub-parts of the
values of 1, 1 and 2 marks each respectively.
7. All Questions are compulsory. However, an internal choice in 2 Qs of 5 marks, 2 Qs of 3 marks
and 2 Questions of 2 marks has been provided. An internal choice has been provided in the
2marks questions of Section E
8. Draw neat figures wherever required. Take π =22/7 wherever required if not stated.
SECTION – A
Questions 1 to 20 carry 1 mark each.
1. If the distance between the points (4,p) and (1,0) is 5, then value of p is
(a) 4 only (b) ±4 (c) -4 only (d) 0
Ans: (b) ±4
The points given are (4, p) and (1, 0)
By distance formula, 5 = √(4 − 1)2 + p2
⇒ 25 = (4 − 1)2 + p2 ⇒ 25 = 32 + p2 ⇒ 25 = 9 + p2
⇒ p2 = 25 – 9 = 16 ⇒ p = ±4
2. If p and q are positive integers such that p = a3b2 and q = a2b, where ‘a’ and ‘b’ are prime
numbers, then the LCM ( p, q) is …..
(a) ab (b) a2b2 (c) a3b2 (d) a3b3
Ans: (c) a3b2

3. 108 can be expressed as a product of its primes as ……………..


(a) 23 x 32 (b) 23 x 33 (c) 22 x 32 (d) 22 x 33
2 3
Ans: (d) 2 x 3
108 = 2 x 2 x 3 x 3 x 3 = 22 x 33

4. Two dice are thrown at the same time and the product of numbers appearing on them is noted.
The probability that the product is a prime number is
(a) 1/3 (b) 1/6 (c) 1/5 (d) 5/6
Ans: (b) 1/6
Total number of possible outcomes = 36
Now for the product of the numbers on the dice is prime number can be have in these possible
ways = (1, 2), (2, 1), (1, 3), (3, 1), (5, 1), (1, 5)
So, number of possible ways = 6
∴ Required probability = 6/36 = 1/6

5. The pair of equations x + 2y + 5 = 0 and –3x – 6y + 1 = 0 have


(a) a unique solution (b) exactly two solutions
(c) Infinitely many solutions (d) no solution
Ans: (d) no solution

Prepared by: M. S. KumarSwamy, TGT(Maths) Page - 1 -


The given equations are:
x + 2y + 5 = 0
−3x − 6y + 1 = 0
From the given equations we have:
a1 1 b 2 1 c 5
 ; 1   ; 1 
a2 3 b2 6 3 c2 1
a b c
 1  1  1
a2 b2 c2
Hence the given pair of equations have no solution.

6. If p and q are the zeroes of the quadratic polynomial f(x) = 2x2 – 7x + 3, find the value of p + q
– pq is
(a) 1 (b) 2 (c) 3 (d) None of these
Ans: (b) 2
Here, a = 2, b = -7 and c = 3
p + q = -b/a = 7/2 and pq = c/a = 3/2
7 3 73 4
p  q  pq     2
2 2 2 2

1
7. If sin 2A = tan² 45° where A is an acute angle, then the value of A is
2
(a) 60° (b) 45° (c) 30° (d) 15°
Ans: (d) 15°
1 1 1
sin 2 A  tan 2 450  12   sin 300  2 A  300  A  150
2 2 2

8. In ΔABC right angled at B, if cotC = √3 , then then cosAsinC + sinAcosC =


(a) –1 (b) 0 (c) 1 (d) √3 / 2
Ans: (c) 1
cot C = √3 = cot 30°, so, ∠C = 30°,
Hence, ∠A = 60°.
1 1 3 3 1 3 1 3 4
So, cos A sin C + sin A cos C         1
2 2 2 2 4 4 4 4

9. If is an acute angle and tan + cot = 2, then the value of sin3 + cos3 is
1 2
(a) 1 (b) (c) (d) 2
2 2
2
Ans: (c)
2
1
tan   cot   2  tan    2  tan 2   2 tan   1  0
tan 
 (tan   1)2  0  tan   1  tan 450    450
3 3
3 3 3 0  1   1 
3 0 1 1 1 2
Now,sin   cos   sin 45  cos 45         
 2  2 2 2 2 2 2 2

10. A girl walks 200m towards East and then 150m towards North. The distance of the girl from the
starting point is
(a) 350m (b) 250m (c) 300 m (d) 325 m
Ans: (b) 250m

Prepared by: M. S. KumarSwamy, TGT(Maths) Page - 2 -


Let girl starting point is A and she goes to B towards east covering 200 m distance .
Now from B she moves 150 m towards north let at point C.

Then, using Pythagoras theorem we get,


⇒ AC = √(AB² + BC²) ⇒ AC = √(200² + 150²) ⇒ AC = √(40000 + 22500)
⇒ AC = √62500 ⇒ AC = 250 m

11. In ABC, DE || AB, If CD = 3 cm, EC = 4 cm, BE = 6 cm, then DA is equal to


(a) 7.5 cm (b) 3 cm (c) 4.5 cm (d) 6 cm
Ans: (c) 4.5 cm
CD CE 3 4 9
DE || AB      DA   4.5cm
DA EB DA 6 2

12. If angle between two radii of a circle is 130⁰, the angle between the tangents at the ends of the
radii is :
(a) 90⁰ (b) 50⁰ (c) 70⁰ (d) 40⁰
Ans: (b) 50⁰

13. The relationship between mean, median and mode for a moderately skewed distribution is
(a) mode = median – 2 mean (b) mode = 3 median – 2 mean
(c) mode = 2 median – 3 mean (d) mode = median – mean
Ans: (b) mode = 3 median – 2 mean

14. For the following distribution:


Marks Below Below Below Below Below Below
10 20 30 40 50 60
No. of Students 3 12 27 57 75 80
the modal class is
(a) 10 – 20 (b) 20 – 30 (c) 30 – 40 (d) 50 – 60
Ans: (c) 30 – 40
Marks 0 – 10 10 – 20 20 – 30 30 – 40 40 – 50 50 – 60
No. of Students 3 9 15 30 18 5
Highest frequency is 30 which belong to 30 – 40. Hence, Modal class is 30 – 40

15. The area of a quadrant of a circle, whose circumference is 22 cm, is


11 2 77 77 77
(a) cm (b) cm2 (c) cm2 (d) cm2
8 8 2 4
77
Ans: (b) cm2
8
Let the radius of the circle be 'r'
Circumference (C) = 22 cm

Prepared by: M. S. KumarSwamy, TGT(Maths) Page - 3 -


⇒ radius (r) = C/2π = 22/(2 × 22/7) = (22 × 7)/(2 × 22) = 7/2 cm
Therefore, the area of a quadrant = 1/4 × πr2
= 1/4 × 22/7 × 7/2 × 7/2
= 77/8 cm2

16. If the quadratic equation x2 + 4x + k = 0 has real and equal roots, then
(a) k < 4 (b) k > 4 (c) k = 4 (d) k ≥ 4
Ans: (c) k = 4
For a quadratic equation to have equal and real roots the discriminant should be equal to zero.
D = 0. Now, D = b2 – 4ac
⇒ 0 = (4)2 – 4 (1)(k) ⇒ 0 = 16 – 4k ⇒ 4k = 16 ⇒ k = 16/4 ⇒ k = 4

17. Volumes of two spheres are in the ratio 64 : 27. The ratio of their surface areas is
(a) 3 : 4 (b) 4 : 3 (c) 9 : 16 (d) 16 : 9
Ans: (d) 16 : 9
Let the radius of two spheres be r1 and r2.
Given, the ratio of the volume of two spheres = 64 : 27
4 3
r
V1 3 1 r 3 64 r 4
  13   1 
V2 4  r 3 r2 27 r2 3
2
3
Let the surface areas of the two spheres be S1 and S2.
2
S1 4 r12 r12  4  16
∴     
S 2 4 r2 2 r2 2  3  9

18. The area of the square that can be inscribed in a circle of radius 8 cm is
(a) 256 cm² (b) 128 cm² (c) 64√2 cm² (d) 64 cm²
Ans: (b) 128 cm²
Radius of circle =8 cm
⇒ Diameter = 8 × 2=16 cm
The diameter of circle = diagonal of square =16 cm = a√2
⇒ a = 16/√2 = 8√2
Area of square = (side)2 = (8√2)2 = 128 cm2

DIRECTION: In the question number 19 and 20, a statement of Assertion (A) is followed by a
statement of Reason (R).
Choose the correct option

19. Assertion (A): If HCF ( 90, 144) = 18, then LCM (90, 144) = 720
Reason (R): HCF (a, b) x LCM (a, b) = a x b
(a) Both Assertion (A) and Reason (R) are true and Reason (R) is the correct explanation of
Assertion (A).
(b) Both Assertion (A) and Reason (R) are true but Reason (R) is not the correct explanation of
Assertion (A).
(c) Assertion (A) is true but Reason (R) is false.
(d) Assertion (A) is false but Reason (R) is true.
Ans: (a) Both Assertion (A) and Reason (R) are true and Reason (R) is the correct
explanation of Assertion (A).
HCF x LCM = Product of two numbers
90  144
 LCM   5  144  720
18

Prepared by: M. S. KumarSwamy, TGT(Maths) Page - 4 -


20. Assertion (A): The point (0, 4) lies on y -axis.
Reason (R): The x co-ordinate on the point on y -axis is zero.
(a) Both Assertion (A) and Reason (R) are true and Reason (R) is the correct explanation of
Assertion (A)
(b) Both assertion (A) and reason (R) are true and reason (R) is not the correct explanation of
Assertion (A)
(c) Assertion (A) is true but reason(R) is false.
(d) Assertion (A) is false but reason(R) is true.
Ans: (a) Both Assertion (A) and Reason (R) are true and Reason (R) is the correct explanation
of Assertion (A)

SECTION – B
Questions 21 to 25 carry 2 marks each.

1
21. If tan (A + B) = 3 and tan (A – B) = ; 0° < A+B ≤ 90°; A > B, find A and B.
3
Ans: tan(A + B) = √3 = tan 60°
⇒ A + B = 60° ……(i)
tan(A – B) = 1/√3 = tan 30°
⇒ A – B = 30° …… (ii)
Adding equation (i) and (ii),
2A = 90° ⇒ A = 45°
Putting the value of A in equation (i),
45° + B = 60°
⇒ B = 60°- 45° ⇒ B = 15°
OR
If xsin3θ + ycos3θ = sinθ cosθ and xsinθ = ysinθ then find x2 + y2.
Ans:

22. ABCD is a trapezium in which AB || CD and its diagonals intersect each other at the point O.
OA OB
Using a similarity criterion of two triangles, show that 
OC OD
Ans: Given : ABCD is a trapezium, AB II CD

In ∆AOB and ∆COD

Prepared by: M. S. KumarSwamy, TGT(Maths) Page - 5 -


∠OBA = ∠ODC ----eq.1 (alt.angles are equal)
∠OAB = ∠OCD ----eq.2 (alt.angles are equal)
Therefore, ∆AOB ∼ ∆COD( A.A Similarity)
OA OB
Hence, 
OC OD

23. The length of the minute hand of a clock is 14 cm. Find the area swept by the minute hand in 5
minutes.
Ans: We know that the minute hand completes one rotation in 1 hour or 60 minutes.
Length of the minute hand (r) = 14 cm
Area swept by minute hand in 1 minute = πr2/60
Thus, area swept by minute hand in 5 minutes = (πr2/60) × 5 = πr2/12
= 1/12 × 22/7 × 14 × 14 cm2
= 154/3 cm2
OR
In a circle of radius 21 cm, an arc subtends an angle of 60° at the centre. Find (i) the length of
the arc (ii) area of the sector formed by the arc
Ans: Here, r = 21 cm, θ = 60°

Area of the segment APB = Area of sector AOPB - Area of ΔAOB


(i) Length of the Arc, APB = θ/360° × 2πr
= 60°/360° × 2 × 22/7 × 21 cm
= 22 cm
(ii) Area of the sector, AOBP = θ/360° x πr2
= 60°/360° × 22/7 × 21 × 21 cm2
= 231 cm2

24. From a point P, two tangents PA and PB are drawn to a circle C(0, r). If OP = 2r, then find
∠APB. Prove that triangle APB is an equilateral triangle.

Ans: Let APO  


OA 1
 sin     sin 300    300
OP 2
 APO  2  2(300 )  600
Also, PAB  PBA  600 ( PA  PB)
⇒ ∆APB is an equilateral triangle.

Prepared by: M. S. KumarSwamy, TGT(Maths) Page - 6 -


25. For what value of k will the following system of linear equations have no solution?
3x + y = 1; (2k – 1) x + (k – 1) y = 2k + 1
Ans: 3x + y – 1 = 0
(2k – 1)x + (k – 1)y – 2k – 1 = 0
a1 3 b1 1 c1 1 1
 ;  ;  
a2 2k  1 b2 k  1 c2 2k  1 2k  1
a b c
For no solutions, 1  1  1
a2 b2 c2
3 1 1 3 1
     ⇒ 3k – 3 = 2k – 1 ⇒ k = 2
2 k  1 k  1 2k  1 2k  1 k  1

SECTION – C
Questions 26 to 31 carry 3 marks each.
26. Prove that √5 is and irrational number.
p
Ans: Let 5 is a rational number then we have 5 , where p and q are co-primes.
q
 p  5q
Squaring both sides, we get p 2  5q 2
 p2 is divisible by 5  p is also divisible by 5
So, assume p = 5m where m is any integer.
Squaring both sides, we get p2 = 25m2
But p 2  5q 2
Therefore, 5q2 = 25m2  q2 = 5m2
 q2 is divisible by 5  q is also divisible by 5
From above we conclude that p and q have one common factor i.e. 5 which contradicts that p
and q are co-primes.
Therefore, our assumption is wrong.
Hence, 5 is an irrational number.

27. Find the zeroes of the quadratic polynomial x2 – 2x – 8 and verify the relationship between the
zeroes and the coefficients of the polynomial.
Ans: Given, f(x)=x2 − 2x – 8
The zeroes of f(x) are given by, f(x) = 0
⇒ x2 + 2x − 4x – 8 = 0 ⇒ x (x + 2) − 4(x + 2) = 0 ⇒ (x + 2) (x − 4) = 0
⇒ x = −2 (or) x = 4
Hence, the zeros of f(x) = x2 − 2x − 8 are α = −2 and β = 4
b
α + β = -2 + 4 = 2 = =2
a
c
αβ = -2 x 4= -8 = =-8
a

28. The sum of the digits of a two-digit number is 9. Also 9 times this number is twice the number
obtained by reversing the order of the digits. Find the number.
Ans: Let the tens digits and unit digit of the number be x and y respectively. Then, the number
will be 10x + y
Number after reversing the digits is 10y + x
According to the question,
x + y = 9... (i)
9(10x + y) = 2(10y + x)
Prepared by: M. S. KumarSwamy, TGT(Maths) Page - 7 -
⇒ 88x – 11y = 0 ⇒ 8x + y = 0... (ii)
Adding equation (i) and (ii), we get
9x = 9 ⇒ x = 1
Putting the value in equation (i), we get y = 8
Hence, the number is 18.
OR
Yash scored 40 marks in a test, getting 3 marks for each right answer and losing 1 mark for each
wrong answer. Had 4 marks been awarded for each correct answer and 2 marks been deducted
for each incorrect answer, then Yash would have scored 50 marks. How many questions were
there in the test?
Ans: Let x be the number of right answers and y be the number of wrong answers.
According to the question,
3x – y = 40 ….(i)
and, 2x – y = 25 ….(ii)
On subtraction, we get: x = 15
putting the value of x in (i), we get 3 (15)-y = 40 y = 5
Number of right answers= 15 answers
Number of wrong answers= 5 answers.
Total Number of questions = 5 + 15 = 20

29. Prove that (sinA + cosecA)2 + (cosA + secA)2 = 7 + tan2A + cot2A


Ans: L.H.S = (sinA + cosecA)2 + (cosA + secA)2
=sin2 A + cosec2 A +2sinAcosecA + cos2 A + sec2 A +2cosAsecA
= sin2 A + cos2 A +cosec2 A+sec2 A +2sinA×1/sinA + 2cosA×1/cosA
Since, (sin2 A + cos2 A =1)
(sec2 A =1 + tan2A, cosec2 A = 1 + cot2A)
= 1 + 1 + cot2A +1 + tan2A +2 +2
= 7 + tan2A +cot2A = RHS

30. Prove that the lengths of the tangents drawn from an external point to a circle are equal.
Ans: Given, To Prove, Constructions and Figure – 1½ marks
Correct Proof – 1½ marks
OR
In the figure XY and X'Y' are two parallel tangents to a circle with centre O and another tangent
AB with point of contact C interesting XY at A and X'Y' at B, what is the measure of ∠AOB.

Ans: Join OC. Since, the tangents drawn to a circle from an external point are equal.
∴ AP = AC

Prepared by: M. S. KumarSwamy, TGT(Maths) Page - 8 -


In Δ PAO and Δ AOC, we have:
AO = AO [Common]
OP = OC [Radii of the same circle]
AP = AC
⇒ Δ PAO ≅ Δ AOC [SSS Congruency]
∴ ∠PAO = ∠CAO = ∠1
∠PAC = 2 ∠1 ...(1)
Similarly ∠CBQ = 2 ∠2 ...(2)
Again, we know that sum of internal angles on the same side of a transversal is 180°.
∴ ∠PAC + ∠CBQ = 180°
⇒ 2 ∠1 + 2 ∠2 = 180° [From (1) and (2)]
⇒ ∠1 + ∠2 = 180°/2 = 90° ...(3)
Also ∠1 + ∠2 + ∠AOB = 180° [Sum of angles of a triangle]
⇒ 90° + ∠AOB = 180°
⇒ ∠AOB = 180° − 90°
⇒ ∠AOB = 90°.

31. Two dice are thrown at the same time. What is the probability that the sum of the two numbers
appearing on the top of the dice is
(i) 8? (ii) 7? (iii) less than or equal to 12?
Ans: (i) Number of outcomes with sum of the numbers 8 = 5
∴ Required Probability = 5/36
(ii) Number of outcomes with sum of the numbers 7 = 6
∴ Required Probability = 6/36 = 1/6
(iii) Number of outcomes with sum of the numbers less than or equal to 12 = 36
∴ Required Probability = 36/36 = 1

SECTION – D
Questions 32 to 35 carry 5 marks each.

32. A person on tour has Rs.360 for his expenses. If he extends his tour for 4 days, he has to cut
down his daily expenses by Rs.3. Find the original duration of the tour.
Ans: Let days be the original duration of the tour.
Total expenditure on tour ₹ 360
Expenditure per day ₹ 360/x
Duration of the extended tour (x + 4) days
Expenditure per day according to the new schedule ₹ 360/(x + 4)
Given that daily expenses are cut down by ₹ 3
360 360
As per the given condition,  3
x x4
1 1 
 360   3
 x x4

Prepared by: M. S. KumarSwamy, TGT(Maths) Page - 9 -


1 1  3 1
   
 x x  4  360 120
x4 x 1 4 1
   
x( x  4) 120 x( x  4) 120
⇒ x(x + 4) = 480
⇒ x² + 4x = 480
⇒ x² + 4x – 480 = 0
⇒ x² + 24x – 20x – 480 = 0
⇒ x(x + 24) - 20(x + 24) = 0
⇒ x – 20 = 0 or x + 24 = 0
⇒ x = 20 or x = -24
Since the number of days cannot be negative. So, x = 20
Therefore, the original duration of the tour was 20 days

OR
Rs.6500 were divided equally among a certain number of persons. Had there been 15 more
persons, each would have got Rs.30 less. Find the original number of persons.
Ans: Let the original number of persons be x
Total money which was divided = Rs. 6500
Each person share = Rs. 6500/x
6500 6500
According to the question,   30
x x  15
6500 x  97500  6500 x
  30
x ( x  15)
97500 3250
  30  1
x ( x  15) x( x  15)
⇒ x² + 15x – 3250 = 0
⇒ x² + 65x – 50x – 3250 = 0
⇒ x(x + 65) – 50(x + 65) = 0
⇒ (x + 65)(x – 50) = 0
⇒ x = -65, 50
Since the number of persons cannot be negative, hence the original numbers of person is 50

33. State and Prove Basic Proportionality Theorem.


Ans: Statement – 1 mark
Given, To prove, Construction and figure of 2 marks
Proof of 2 marks

34. Ramesh made a bird-bath for his garden in the shape of a cylinder with a hemispherical
depression at one end. The height of the cylinder is 1.45 m and its radius is 30 cm. Find the total
surface area of the bird-bath.

Prepared by: M. S. KumarSwamy, TGT(Maths) Page - 10 -


Ans: Let h be height of the cylinder, and r the common radius of the cylinder and hemisphere.
Then, the total surface area = CSA of cylinder + CSA of hemisphere
= 2 rh + 2 r2 = 2 r (h + r)
22
=2x x 30 (145 + 30) cm2
7
22
=2x x 30 x 175 cm2
7
= 33000 cm2 = 3.3 m2
OR
A tent is in shape of a cylinder surmounted by a conical top. If the height and diameter of the
cylindrical part are 2.1m and 4m respectively and the slant height of the top is 2.8m. Find the
area of canvas used for making the tent. Also find the cost of canvas of the tent at the rate of 500
per m2.
Ans: Radius = 2m, Slant height l= 2.8m , height h= 2.1m
Cost of canvas per m2= Rs.500
Area of canvas used = CSA of cone + CSA of cylinder
= πrl + 2πrh
=22/7 x 2 x 2.8 + 2 x 22/7 x 2 x 2.1
=17.6 + 26.4
=44m2
Cost of the canvas of tent =44 x 500
=Rs.22,000

35. The following frequency distribution gives the monthly consumption of 68 consumers of a
locality. Find median, mean and mode of the data and compare them.
Monthly consumption of Number of consumers
electricity (in units)
65-85 4
85-105 5
105-125 13
125-145 20
145-165 14
165-185 8
185-205 4
Ans: For mean , median , mode
To calculate xi , cumulative frequency , identifying highest frequency
Formulae for mean , median, mode

Mean = a   i i  135 
fd 140
 fi 68
=137.05
n 
 2  cf   34  22 
Median = l    h  = 125    20   125  12  137
 f   20 
 
 f1  f 0   20  13 
Mode = l    h   125    20 
 2 f1  f 0  f 2   40  13  14 
7 
 125    20   125  10.77  135.77
 13 

Prepared by: M. S. KumarSwamy, TGT(Maths) Page - 11 -


SECTION – E(Case Study Based Questions)
Questions 36 to 38 carry 4 marks each.

36. Case Study-1


Mohan takes a loan from a bank for his car. Mohan replays his total loan of Rs.118000 by
paying every month starting with the first instalment of Rs.1000. If he increases the instalment
by Rs.100 every month.

(i) What is the first term and common difference of given question. (1)
(ii) The amount paid buy him in 30th installment. (1)
(iii) The amount paid by him in the 30 installments is (2)
(OR)
(iii) What amount does he still have to pay after 30th installment? (2)
Ans: (i) Given , a1 =1000
Common difference, d =100
Total loan= Rs.1,18,000
(ii)a30 = a + 29d
= 1000 + 29 x 100
= 3900
Amount paid in 30th installment is Rs.3900
30
(iii) S30 = [2x 1000 +(30 -1) x 100]
2
=15 x 4900
=73,500
Amount paid in 30 installments is Rs.73,500
(OR)
The amount he still have to pay after 30 installments=Rs.118000 – Rs. 73,500
=Rs.44,500

37. Case Study-2


In order to conduct sports day activities in your school, lines have been drawn with chalk
powder at a distance of 1 m each in a rectangular shaped ground ABCD. 100 flower pots have
been placed at the distance of 1 m from each other along AD, as shown in the following figure.
1 1
Niharika runs ( )th distance AD on the 2nd line and posts a green Flag. Preet runs ( ) th
4 5
distance AD on the eighth line and posts are red flags. Taking A as the origin AB along x-axis
and AD along y-axis, answer the following questions:
(i) Find the coordinates of the green flag. (1)
(ii) Find the distance between the two flags. (1)
(iii) If Rashmi has to post a blue flag exactly halfway between the line segment joining the two
flags, where should she post her flag? (2)
OR

Prepared by: M. S. KumarSwamy, TGT(Maths) Page - 12 -


(iii) If Joy has to post a flag at one fourth distance from the green flag, in the line segment
joining the green and red flags, then where should he post his flag? (2)

 1 
Ans: (i) Position of the red flag is  2, 100   (2, 25)
 4 
(ii) Distance between the two flags = (36  25) = 61cm
 2  8 25  20 
(iii) Position of the blue flag =  , 
 2 2 
  5, 22.5 
OR

Required point is (3.5, 23.75)

38. Case Study – 3


Lakshaman Jhula is located 5 kilometers north-east of the city of Rishikesh in the Indian state of
Uttarakhand. The bridge connects the villages of Tapovan to Jonk. Tapovan is in Tehri Garhwal
district, on the west bank of the river, while Jonk is in Pauri Garhwal district, on the east bank.
Lakshman Jhula is a pedestrian bridge also used by motorbikes. It is a landmark of Rishikesh. A
group of Class X students visited Rishikesh in Uttarakhand on a trip. They observed from a
point (P) on a river bridge that the angles of depression of opposite banks of the river are 60°
and 30° respectively. The height of the bridge is about 18 meters from the river.

Prepared by: M. S. KumarSwamy, TGT(Maths) Page - 13 -


Based on the above information answer the following questions.
(i) Find the distance PA. (1)
(ii) Find the distance PB (1)
(iii) Find the width AB of the river. (2)
OR
(iii) Find the height BQ if the angle of the elevation from P to Q be 30°. (2)
PC
Ans: (i) sin 600 
PA
3 18 36
   PA   12 3m
2 PA 3
PC
(ii) sin 300 
PB
1 18
   PB  36 m
2 PB
PC 18
(iii) tan 60 0   3  AC  6 3m
AC AC
PC 1 18
tan 300     CB  18 3m
CB 3 CB
Width AB = AC + CB = 6 3m  18 3m  24 3m
OR
RB = PC = 18 m & PR = CB = 18√3m
QR 1 QR
tan 300     QR  18m
PR 3 18 3
QB = QR + RB = 18 + 18 = 36 m.
Hence height BQ is 36m.

Prepared by: M. S. KumarSwamy, TGT(Maths) Page - 14 -


KENDRIYA VIDYALAYA GACHIBOWLI, GPRA CAMPUS, HYD-32
SAMPLE PAPER TEST 06 FOR BOARD EXAM 2023
(ANSWERS)
SUBJECT: MATHEMATICS MAX. MARKS : 80
CLASS : X DURATION : 3 HRS
General Instruction:
1. This Question Paper has 5 Sections A-E.
2. Section A has 20 MCQs carrying 1 mark each.
3. Section B has 5 questions carrying 02 marks each.
4. Section C has 6 questions carrying 03 marks each.
5. Section D has 4 questions carrying 05 marks each.
6. Section E has 3 case based integrated units of assessment (04 marks each) with sub-parts of the
values of 1, 1 and 2 marks each respectively.
7. All Questions are compulsory. However, an internal choice in 2 Qs of 5 marks, 2 Qs of 3 marks
and 2 Questions of 2 marks has been provided. An internal choice has been provided in the 2marks
questions of Section E
8. Draw neat figures wherever required. Take π =22/7 wherever required if not stated.
SECTION – A
Questions 1 to 20 carry 1 mark each.

1. If the LCM of a and 18 is 36 and the HCF of a and 18 is 2, then a =


(a) 1 (b) 2 (c) 3 (d) 4
Ans. (d) 4

2. The sum of exponents of prime factors in the prime-factorisation of 196 is:


(a) 3 (b) 4 (c) 5 (d) 6
Ans. (b) 4

3. If the zeroes of the quadratic polynomial x2 + (a + 1)x + b are 2 and -3, then
(a) a = -7, b = -1 (b) a = 5, b = -1 (c) a = 2, b = –6 (d) a = 0, b = –6
Ans. (d) a = 0, b = –6

4. Three cubes each of side 15 cm are joined end to end. The total surface area of the cuboid is:
(a) 3150 cm2 (b) 1575 cm2 (c) 1012.5 cm2 (d) 576.4 cm2
Ans. (a) 3150 cm2

5. The point which lies on the perpendicular bisector of the line segment joining point A (–2, –5)
and B (2, 5) is:
(a) (0, 0) (b) (0, –1) (c) (–1, 0) (d) (1, 0)
Ans. (a) (0, 0)

6. The point on the x-axis which is equidistant from (– 4, 0) and (10, 0) is:
(a) (7, 0) (b) (5, 0) (c) (0, 0) (d) (3, 0)
Ans. (d) (3, 0)

7. If x = 2sin2θ and y = 2cos2θ + 1 then x + y is:


(a) 3 (b) 2 (c) 1 (d) 1/2
Ans: (a) 3

8. If cos θ + cos2 θ = 1, the value of sin2 θ + sin4 θ is :


(a) –1 (b) 0 (c) 1 (d) 2

Prepared by: M. S. KumarSwamy, TGT(Maths) Page - 1 -


Ans: (c) 1

9. In the figure given below, AD = 4 cm, BD = 3 cm and CB = 12 cm, then cot θ equals :

(a) 3/4 (b) 5/12 (c) 4/3 (d) 12/5


Ans: (d) 12/5

10. The perimeters of two similar triangles are 26 cm and 39 cm. The ratio of their areas will be :
(a) 2 : 3 (b) 6 : 9 (c) 4 : 6 (d) 4 : 9
Ans: (d) 4 : 9

11. If ∆ABC ~ ∆EDF and ∆ABC is not similar to ∆DEF, then which of the following is not true?
(a) BC.EF = AC.FD (b) AB.EF = AC.DE (c) BC.DE = AB.EF (d) BC.DE = AB.FD
Ans. (c) BC.DE = AB.EF

12. In the given figure, from an external point P, two tangents PQ and PR are drawn to a circle of
radius 4 cm with centre O. If ∠QPR = 90°, then length of PQ is

(a) 3 cm (b) 4 cm (c) 2 cm (d) 2.2 cm


Ans: (b) 4 cm

13. In a circle of diameter 42cm, if an arc subtends an angle of 60º at the centre, then the length of the
arc is:
(a) 22/7 cm (b) 11cm (c) 22 cm (d) 44 cm
Ans: (c) 22 cm

14. If the circumference of a circle increases from 2π to 4π then its area ......the original area :
(a) Half (b) Double (c) Three times (d) Four times
Ans: (d) Four times

15. The radii of 2 cylinders are in the ratio 2 : 3 and their heights are in the ratio 5 : 3. Then, the ratio
of their volumes is:
(a) 19 : 20 (b) 20 : 27 (c) 18:25 (d) 17:23
Ans: (b) 20 : 27

16. Consider the following frequency distribution


Class 0–5 6 – 11 12 – 17 18 – 23 24 – 29
Frequency 13 10 15 8 11
The upper limit of the median class is
(a) 7 (b) 17.5 (c) 18 (d) 18.5

Prepared by: M. S. KumarSwamy, TGT(Maths) Page - 2 -


Ans: (b) 17.5

17. For the following distribution:


Marks Below 10 Below 20 Below 30 Below 40 Below 50 Below 60
No. of students 3 12 27 57 75 80
the modal class is
(a) 10-20 (b) 20-30 (c) 30-40 (d) 50-60
Ans: (c) 30-40

18. A box contains cards numbered 6 to 50. A card is drawn at random from the box. The probability
that the drawn card has a number which is a perfect square is :
(a) 1/45 (b) 2/15 (c) 4/45 (d) 1/9
Ans. (d) 1/9
P(perfect Square)= 5/45 = 1/9

Direction : In the question number 19 & 20 , A statement of Assertion (A) is followed by a


statement of Reason(R) . Choose the correct option
19. Assertion (A): The mid-point of the line segment joining the points A (3, 4) and B (k, 6) is P (x,
y) and x + y – 10 = 0, the value of k is 7
 x  x y  y2 
Reason (R): Midpoint of line segment is  1 2 , 1 
 2 2 
(a) Both A and R are true and R is the correct explanation of A
(b) Both A and R are true but R is not the correct explanation of A
(c) A is true and R is false
(d) A is false and R is true
Ans: (b) Both A and R are true but R is not the correct explanation of A
20. Assertion (A): The largest number that divide 70 and125 which leaves remainder 5 and 8 is 13
Reason (R): HCF (65,117) =13
(a) Both A and R are true and R is the correct explanation of A
(b) Both A and R are true but R is not the correct explanation of A

(c) A is true and R is false


(d) A is false and R is true
Ans: (b) Both A and R are true but R is not the correct explanation of A

SECTION-B
Questions 21 to 25 carry 2M each

21. The short and long hands of a clock are 4 cm and 6 cm long respectively. Find the sum of
distances travelled by their tips in 2 days
Ans: In 2 days, the short hand will complete 4 rounds.
∴ Distance moved by its tip = 4(circumference of a circle of radius 4 cm)
 22  704
 4   2   4  cm  cm
 7  7
In 2 days, the long hand will complete 48 rounds.
∴ Distance moved by its tip = 48(circumference of a circle of radius 6 cm)
 22  12.672
 48   2   6  cm  cm
 7  7
 704 12672 
Hence, sum of distances moved by the tips of two hands of the clock     cm
 7 7 
= 1910.85 cm

Prepared by: M. S. KumarSwamy, TGT(Maths) Page - 3 -


OR
A car has two wipers which do not overlap. Each wiper has a blade of length 21 cm sweeping
through an angle of 120°. Find the total area cleaned at each sweep of the blades
Ans: Here, r = 21 cm, θ = 120°
 2 1200 22
Area of a sector =   r    21 21
3600 3600 7
= 462 cm2
∴ Total area cleaned by two wipers
= 2 × 462 = 924 cm2
22. A quadrilateral ABCD is drawn to circumscribe a circle. Prove that AB + CD = AD + BC.

Ans: We know that the lengths of the tangents drawn from an external point to the circle
are equal.
DR = DS ...... (i)
BP = BQ ...... (ii)
AP = AS ...... (iii)
CR = CQ ...... (iv)
Adding (i), (ii), (iii), (iv), we get DR + BP + AP + CR = DS + BQ + AS + CQ
By rearranging the terms we get,
(DR + CR) + (BP + AP) = (CQ + BQ) + (DS + AS)
⇒ CD + AB = BC + AD
Hence it is proved AB + CD = AD + BC.

1 1
23. If sin(A – B) = , cos(A + B) = , 00< A + B ≤900 , A > B. Find A and B.
2 2
1  1
Ans: sin( A  B)   sin( A  B)  30  sin 30  
2  2
On equating both sides
A  B  30 (1)
1  1
cos( A  B)   cos( A  B )  cos(60 )  cos(60 )  
2  2
On equating both sides
A  B  60..(2)
Adding (1) and (2),we get 2A = 900 ⇒ A = 450
Putting value of A in (i)
450 + B = 600 ⇒ B = 150

24. Find the value of m for which the pair of linear equations:
2x + 3y – 7 = 0 and (m – 1) x + (m + 1) y = (3m – 1) has infinitely many solutions
Ans: For infinitely many solutions the condition is
a1 b1 c1 2 3 7
    
a2 b2 c2 m  1 m  1 3m  1
Now, 2(m + 1) = 3(m – 1) ⇒ m = 5

Prepared by: M. S. KumarSwamy, TGT(Maths) Page - 4 -


and 3(3m – 1) = 7(m + 1) ⇒ m = 5
Hence, for m = 5, the system has infinitely many solutions.

QR QT
25. In the figure,  and ∠1 = ∠2, Show that ∆PQS ∼ ∆TQR.
QS PR

Ans: In △PQR,
Since, ∠1=∠2
∴ PR=PQ (Opposite sides of equal angles are equal) .....(1)
QR QT
In △PQS and △TQR,  . (Given)
QS PR
QR QT
i.e.,  .( From 1)
QS PQ
Also, ∠Q is common
∴ By SAS criterion of similarity, △PQS∼△TQR.

OR
ABCD is a trapezium in which AB || CD and its diagonals intersect each other at the point O.
Using a similarity criterion of two triangles, show that =
Ans: ABCD is a trapezium with AB∥CD and diagonals AB and CD intersecting at O.

In △OAB and △OCD


∠AOB =∠DOC [ Vertically opposite angles ]
∠ABO = ∠CDO [ Alternate angles ]
∠BAO = ∠OCD [ Alternate angles ]
∴ △OAB ∼ △OCD [ AAA similarity ]
We know that if triangles are similar, their corresponding sides are in proportional
OA OB
 
OC OD

SECTION-C
Questions 26 to 31 carry 3 marks each

26. On a morning walk, three persons step off together and their steps measure 40 cm, 42 cm and 45
cm, respectively. Find the minimum distance each should walk so that each can cover the same
distance in complete steps.
Ans: The minimum distance each should walk so that each can cover the same distance in
complete steps is the LCM of 40 cm, 42 cm and 45 cm.
Prime factorisation of 40, 42 and 45 gives

Prepared by: M. S. KumarSwamy, TGT(Maths) Page - 5 -


40 = 23 × 5, 42 = 2 × 3 × 7, 45 = 32 × 5
LCM (40, 42, 45) = Product of the greatest power of each prime factor involved in the numbers
= 23 × 32 × 5 × 7 = 8 × 9 × 35 = 72 × 35 = 2520 cm.

27. If a, b are the zeroes of the polynomial 2x2 – 5x + 7, then find a polynomial whose zeroes are 2a
+ 3b, 3a + 2b
Ans: Since a, b are the zeroes of 2x2 – 5x + 7
(5) 5 7
∴a+b= = and ab =
2 2 2
The given zeroes of required polynomial are 2a + 3b and 3a + 2b
5 25
Sum of the zeroes = 2a + 3b + 3a + 2b = 5a + 5b = 5(a + b) = 5 × =
2 2
Again, product of the zeroes = (2a + 3b) (3a + 2b) = 6 (a2 + b2) + 13ab
= 6 [(a + b)2 – 2ab)] + 13 ab = 6(a + b)2 + ab
= 6(5/2)2 + 7/2 = 75/2 + 7/2 = 82/2 = 41
Now, required polynomial is given by
25
k [x2 – (Sum of the zeroes) x + Product of the zeroes] = k [ x2 – x + 41]
2
k
= [2 x 2  25 x  82] , where k is any non-zero real number.
2
Hence the required polynomial is 2 x 2  25 x  82 .

28. Prove that (sinA + cosecA)2 + (cosA + secA)2 = 7 + tan2A + cot2A


Ans: L.H.S. =(sinA + cosecA)2 + (cosA + secA)2
= sin2A + cosec2A + 2sinAcosecA + cos2A + sec2A + 2cosAsecA
= 1 + 2 + 2 + 1 + tan2A + 1 + cot2A
= 7 + tan2A + cot2A = R.H.S.
OR
cos A 1  sin A
Prove that   2sec A
1  sin A cos A
cos A 1  sin A
Ans: LHS  
1  sin A cos A
2 2
cos A  (1  sin A) cos 2 A  1  sin 2 A  2sin A
 
cos A(1  sin A) cos A(1  sin A)
2  2sin A 2
   2sec A  RHS
cos A(1  sin A) cos A

29. Find the ratio in which the line 2x + y – 4 = 0 divides the line segment joining the points A (2, –
2) and B (3, 7)
Ans: Let P(x, y) be the point on the line 2x + y – 4 = 0 dividing the line segment joining the
points A(2, –2) and B(3, 7) in the ratio k : 1.
 3k  2 7 k  2 
∴ The coordinate of P are  , 
 k 1 k 1 
Since, point (x, y) lies on the line 2x + y = 4.
 3k  2   7k  2  6k  4  7k  2
 2  4 4
 k 1   k 1  k 1
⇒ 13k + 2 = 4k + 4 ⇒ 9k = 2 ⇒ k = 2/9
Thus, required ratio is 2 : 9.

Prepared by: M. S. KumarSwamy, TGT(Maths) Page - 6 -


30. In the given figure, OP is equal to diameter of the circle. Prove that ABP is an equilateral triangle.

Ans: Join OP and let it meets the circle at point Q.


Since OP = 2r (Diameter of the circle)
⇒ OQ = QP = r
Consider ΔAOP in which OA ⊥ AP and OP is the hypotenuse.
∴ OQ = AQ = OA
(Mid-point of the hypotenuse is equidistant from the vertices)
⇒ OAQ is an equilateral triangle.
⇒ ∠AOQ = 60° (Each angle of an equilateral triangle is 60°)

Consider right-angled triangle OAP.


∠AOQ = 60° (Proved above)
∠OAP = 90° ⇒ ∠APO = 30°
∠APB = 2∠APO = 2 × 30° = 60°
Also PA = PB (Tangents to a circle from an external point are equal.)
⇒ ∠PAB = ∠PBA (Angles opposite to equal sides in ΔPAB)
In ΔABP, ∠APB = 60°
1800  600
⇒ ∠PAB = ∠PBA =  600
2
⇒ Each angle of DPAB is 60°
⇒ PAB is an equilateral triangle.
OR
A circle is inscribed in a ΔABC having sides 8 cm, 10 cm and 12 cm as shown in the following
figure. Find AD, BE and CF.

Ans: Let AD = x1, BE = x2 and CF = x3;


then AF = AD = x1, BD = BE = x2
and CE = CF = x3.
∴ x1 + x2 = 12; x2 + x3 = 8; x1 + x3 = 10 (1)
Adding,

Prepared by: M. S. KumarSwamy, TGT(Maths) Page - 7 -


2(x1 + x2 + x3) = 30
⇒ x1 + x2 + x3 = 15
Solve for x1, x2 and x3 to get
AD = 7 cm, BE = 5 cm, CF = 3 cm

31. From a pack of 52 playing cards, jacks, queens, kings and aces of red colour are removed. From
the remaining a card is drawn at random. Find the probability that the card drawn is (i) a black
queen (ii) a red card (iii) a face card.
Ans: From the total playing 52 cards, red coloured jacks, queen, kings and aces are removed(i.e.,
2 jacks, 2 queens, 2 kings, 2 aces) ∴ Remaining cards = 52 – 8 = 44
(i) Favourable cases for a black queen are 2 (i.e., queen of club or spade)
∴ Probability of drawing a black queen = 2/44 = 1/22
(ii) Favourable cases for red cards are 26 – 8 = 18 (as 8 cards have been removed) (i.e.9 diamonds
+ 9 hearts)
∴ Probability of drawing a red card = 18/44 = 9/22
(iii) Favourable cases for a face card are 6 (i.e. 2 black jacks, queens and kings each)
∴ Probability of drawing a face card = 6/44 = 3/22

SECTION-D
Questions 32 to 35 carry 5M each

32. A survey regarding the heights (in cm) of 50 girls of class Xth of a school was conducted
and the following data was obtained. Find the mean, median and mode of the given data.
Heights (in cm) 120 – 130 130 – 140 140 – 150 150 – 160 160 – 170
No. of Girls 2 8 12 20 8
Ans:

33. 200 logs are stacked in the following manner: 20 logs in the bottom row, 19 in the next row, 18 in
the row next to it and so on (see below figure). In how may rows are the 200 logs placed and how
many logs are in the top row?
Ans: Here, a is the first term, d is a common difference and n is the number of terms.
It can be observed that the number of logs in rows are forming an A.P. 20, 19, 18, ...
n
We know that sum of n terms of AP is given by the formula Sₙ = [2a + (n - 1) d]
2

Prepared by: M. S. KumarSwamy, TGT(Maths) Page - 8 -


n
⇒ 200 = [2 × 20 + (n - 1)(- 1)] ⇒ 400 = n [40 - n + 1] ⇒ 400 = n [41 - n]
2
⇒ 400 = 41n - n² ⇒ n² - 41n + 400 = 0 ⇒ n² - 16n - 25n + 400 = 0
⇒ n(n - 16) - 25(n - 16) = 0 ⇒ (n - 16)(n - 25) = 0
⇒ Either (n -16) = 0 or (n - 25) = 0
∴ n = 16 or n = 25
The number of logs in nth row will be aₙ = a + (n - 1) d
⇒ a₁₆ = a + 15d ⇒ a₁₆ = 20 + 15 × (- 1) ⇒ a₁₆ = 20 – 15 ⇒ a₁₆ = 5
Similarly, a₂₅ = 20 + 24 × (- 1)
⇒ a₂₅ = 20 – 24 ⇒ a₂₅ = - 4
Clearly, the number of logs in the 16th row is 5. However, the number of logs in the 25th row is
negative 4, which is not possible.
Therefore, 200 logs can be placed in 16 rows. The number of logs in the top (16th) row is 5.
OR
The sum of the third and the seventh terms of an AP is 6 and their product is 8. Find the sum of
first sixteen terms of the AP.
Ans: Here, a is the first term, d is the common difference and n is the number of terms.
Given: a₃ + a₇ = 6 ----- (1)
a₃ × a₇ = 8 ----- (2)
We know that nth term of AP is aₙ = a + (n - 1)d
Third term, a₃ = a + 2d ----- (3)
Seventh term, a₇ = a + 6d ----- (4)
Using equation (3) and equation (4) in equation (1) to find the sum of the terms,
(a + 2d) + (a + 6d) = 6
⇒ 2a + 8d = 6 ⇒ a + 4d = 3 ⇒ a = 3 - 4d ----- (5)
Using equation (3) and equation (4) in equation (2) to find the product of the terms,
(a + 2d ) × (a + 6d ) = 8
Substituting the value of a from equation (5) above,
(3 - 4d + 2d) × (3 - 4d + 6d) = 8
⇒ (3 - 2d) × (3 + 2d) = 8
⇒ (3)² - (2d)² = 8 [Since (a + b)(a - b) = a² - b² ]
⇒ 9 - 4d² = 8 ⇒ 4d² = 1 ⇒ d² = ¼ ⇒ d = ½, -½
Case 1: When d = ½
a = 3 - 4d = 3 - 4 × ½ = 3 - 2 = 1
n 16
Sₙ = [2a + (n - 1) d] ⇒ S₁₆ = [ 2 × 1 + (16 - 1) × ½ ] = 8 × 19/2 = 76
2 2
Case 2: When d = - ½
a = 3 - 4d = 3 - 4 × (- ½) = 3 + 2 = 5
n 16
Sₙ = [2a + (n – 1) d] ⇒ S₁₆ = [2 × 5 + (16 - 1) × (- ½)] = 8 [10 - 15/2] = 8 × 5/2 = 20
2 2

34. Prove that if a line is a drawn parallel to one side of a triangle intersecting the other two sides in
distinct points, then the other two sides are divided in the same ratio. Using the above theorem.
Prove that = if LM II CB and LNII CD as shown in the figure.

Ans: Given, To Prove, Figure and Construction – 1 ½ marks

Prepared by: M. S. KumarSwamy, TGT(Maths) Page - 9 -


Proof - 1 ½ mark
In △ABC, LM ∥ BC
AM AL
∴ By proportionality theorem,  .(1)
AB AC
Similarly, In △ADC, LN ∥ CD
AN AL
∴ By proportionality theorem,   (2)
AD AC
AM AN
∴ from (1) and (2), 
AB AD

1
35. Two pipes running together can fill a cistern in 3 hours. If one pipe takes 3 hours more than
13
the other to fill it, find the time in which each pipe would fill the cistern.
Ans: Let time taken by faster pipe to fill the cistern be x hrs.
Therefore, time taken by slower pipe to fill the cistern = (x + 3) hrs
Since the faster pipe takes x minutes to fill the cistern.
1
∴ Portion of the cistern filled by the faster pipe in one hour =
x
1
Portion of the cistern filled by the slower pipe in one hour =
x3
1 13
Portion of the cistern filled by the two pipes together in one hour = 
40 40
13
1 1 13 x  3  x 13
According to question,    
x x  3 40 x( x  3) 40
⇒ 40 (2x + 3) = 13x (x + 3) ⇒ 80x + 120 = 13x2 + 39x
⇒ 13x2 – 41x – 120 = 0 ⇒ 13x2 – 65x + 24x – 120 = 0
⇒ 13x (x – 5) + 24 (x – 5) = 0 ⇒ (x – 5) (13x + 24) = 0
Either x – 5 = 0 or 13x + 24 = 0
⇒ x = 5 as x = −24/13 not possible.
Hence, the time taken by the two pipes is 5 hours and 8 hours respectively.
OR
If Zeba was younger by 5 years than what she really is, then the square of her age (in years)
would have been 11 more than five times her actual age. What is her age now? [NCERT
Exemplar]
Ans: Let the present age of Zeba be x years.
Age before 5 years = (x – 5) years
According to given condition, (x – 5)2 = 5x + 11
⇒ x2 + 25 – 10x = 5x + 11 ⇒ x2 – 10x – 5x + 25 – 11 = 0
⇒ x2 – 15x + 14 = 0 ⇒ x2 – 14x – x + 14 = 0
⇒ x (x – 14) – 1 (x – 14) = 0 ⇒ (x – 1) (x – 14) = 0
⇒ x – 1 = 0 or x – 14 = 0
⇒ x = 1 or x = 14
But present age cannot be 1 year.
Hence, Present age of Zeba is 14 years.

SECTION-E (Case Study Based Questions)


Questions 36 to 38 carry 4M each

36. On the roadway, Points A and B, which stand in for Chandigarh and Kurukshetra, respectively,
are located nearly 90 kilometres apart. At the same time, a car departs from Kurukshetra and one
from Chandigarh. These cars will collide in 9 hours if they are travelling in the same direction,

Prepared by: M. S. KumarSwamy, TGT(Maths) Page - 10 -


and in 9/7 hours if they are travelling in the other direction. Let X and Y be two cars that are
travelling at x and y kilometres per hour from places A and B, respectively. On the basis of the
above information, answer the following questions:

(a) When both cars move in the same direction, then find the situation can be represented
algebraically. [2]
OR
(a) When both cars move in the opposite direction, then find the situation can be represented
algebraically. [2]
(b) Find the speed of car x. [1]
(c) Find the speed of car y. [1]
Ans: (a) Suppose two cars meet at point Q. Then, Distance travelled by car X = AQ, Distance
travelled by car Y = BQ. It is given that two cars meet in 9 hours.
∴ Distance travelled by car X in 9 hours = 9x km = AQ = 9x
Distance travelled by car Y in 9 hours = 9y km = BQ = 9y
Clearly, AQ - BQ = AB = 9x - 9y = 90 = x - y = 10
OR
Suppose two cars meet at point P. Then Distance travelled by car X = AP and Distance travelled
by car Y = BP.
9 9
In this case, two cars meet in hours. Distance travelled by car X in hours
7 7
9 9
= x km ⇒ AP = x
7 7
9
Distance travelled by car Y in hours
7
9 9
= y km ⇒ BP = y
7 7
Clearly, AP + BP = AB
9 9 9
⇒ x + y = 90 ⇒ (x + y) = 90 ⇒ x + y = 70
7 7 7
(b) We have x - y = 10 and x + y = 70
Adding equations (i) and (ii), we get 2x = 80 ⇒ x = 40
Hence, speed of car X is 40 km/hr.
(c) We have x - y = 10 ⇒ 40 - y = 10 ⇒ y = 30
Hence, speed of car y is 30 km/hr

37. In a toys manufacturing company, wooden parts are assembled and painted to prepare a toy. One
specific toy is in the shape of a cone mounted on a cylinder. For the wood processing activity
center, the wood is taken out of storage to be sawed, after which it undergoes rough polishing,
then is cut, drilled and has holes punched in it. It is then fine polished using sandpaper. For the
retail packaging and delivery activity center, the polished wood sub-parts are assembled together,
then decorated using paint. The total height of the toy is 26 cm and the height of its conical part is
6 cm. The diameters of the base of the conical part is 5 cm and that of the cylindrical part is 3 cm.
On the basis of the above information, answer the following questions:

Prepared by: M. S. KumarSwamy, TGT(Maths) Page - 11 -


(a) If its cylindrical part is to be painted yellow, find the surface area need to be painted. [1]
(b) If its conical part is to be painted green, find the surface area need to be painted. [2]
OR
(b) Find the volume of the wood used in making this toy. [2]
(c) If the cost of painting the toy is 3 paise per sq cm, then find the cost of painting the toy. (Use π
= 3.14) [1]
Ans: Let the radius of cone be r, slant height of cone be l, height of cone be h, radius of cylinder
be r′ and height of cylinder be h′.
Then r = 2.5 cm, h = 6 cm, r′ = 1.5 cm, h′ = 26 – 6 = 20 cm and
Slant height, l  r 2  h 2  2.52  62  6.25  36  42.25  6.5cm
(a) Area to be painted yellow = CSA of the cylinder + area of one base of the cylinder
= 2πr′h′ + π(r′)2 = πr′ (2h′ + r′) = (3.14 × 1.5) (2 × 20 + 1.5) cm2
= 4.71 × 41.5 cm2
= 195.465 cm2
(b) Area to be painted green = CSA of the cone + base area of the cone – base area of the cylinder
= πrl + πr2 – π(r′)2 = π[(2.5 × 6.5) + (2.5)2 – (1.5)2] cm2
= π[20.25] cm2 = 3.14 × 20.25 cm2
= 63.585 cm2
OR
Volume of wood used in making the toy = Volume of cone + Volume of cylinder
1 1  1 
=  r 2 h   r '2 h '    r 2 h  r '2 h '  3.14   2.5  2.5  6  1.5  1.5  20 
3 3  3 
3
 3.14(12.5  4.5)  53.38cm
(c) Total area of painting = 195.465 + 63.585 = 259.05 cm2
Cost of painting 1 cm2 = Re. 0.03
Total cost of painting = Rs. 0.03 x 256.05
= Rs. 7.77

38. Radio towers are used for transmitting a range of communication services including radio and
television. The tower will either act as an antenna itself or support one or more antennas on its
structure, including microwave dishes. They are among the tallest human-made structures. There
are 2 main types: guyed and self-supporting structures. On a similar concept, a radio station tower
was built in two sections A and B.
Tower is supported by wires from a point O. Distance between the base of the tower and point O
is 36 m. From point O, the angle of elevation of the top of section B is 30° and the angle of
elevation of the top of section A is 45°.

Prepared by: M. S. KumarSwamy, TGT(Maths) Page - 12 -


(i) What is the height of the section B? (1)
(ii) What is the height of the section A? (1)
(iii) What is the length of the wire structure from the point O to the top of section A? (2)
OR
(iii) What is the length of the wire structure from the point O to the top of section B? (2)
Ans: Given, that the distance between the base of the tower and point O = 36 m
BC
(i)Consider ΔOCB, tan 300 = ⇒ =
OC
Hence, BC= 12√3=20.78 m
AB  BC
(ii)In ΔOAC, tan 450 = ⇒ = 1 ⇒ AC = 36m
OC
(iii) length of the wire structure from the point O to the top of the section
cos 450 = ⇒ OA = 36√2M
OR
length of the wire structure from the point O to the top of the section
cos 300 = ⇒ = ⇒ OB = 72 / √3 = 24√3m

Prepared by: M. S. KumarSwamy, TGT(Maths) Page - 13 -


KENDRIYA VIDYALAYA GACHIBOWLI, GPRA CAMPUS, HYD-32
SAMPLE PAPER TEST 07 FOR BOARD EXAM 2023
(ANSWERS)
SUBJECT: MATHEMATICS MAX. MARKS : 80
CLASS : X DURATION : 3 HRS
General Instruction:
1. This Question Paper has 5 Sections A-E.
2. Section A has 20 MCQs carrying 1 mark each.
3. Section B has 5 questions carrying 02 marks each.
4. Section C has 6 questions carrying 03 marks each.
5. Section D has 4 questions carrying 05 marks each.
6. Section E has 3 case based integrated units of assessment (04 marks each) with sub-parts of the
values of 1, 1 and 2 marks each respectively.
7. All Questions are compulsory. However, an internal choice in 2 Qs of 5 marks, 2 Qs of 3 marks and
2 Questions of 2 marks has been provided. An internal choice has been provided in the 2marks
questions of Section E
8. Draw neat figures wherever required. Take π =22/7 wherever required if not stated.
SECTION – A
Questions 1 to 20 carry 1 mark each.

1. If p and q are positive integers such that p = a3b2 and q = a2b, where ‘a’ and ‘b’ are prime numbers,
then the HCF ( p, q) is …..
(a) a2b (b) a2b2 (c) a3b2 (d) a3b3
2
Ans: (a) a b

2. ∆ABC ~ ∆PQR. Area of ∆ABC = 81 cm2 and area of ∆PQR = 121 cm2. If altitude AD = 9 cm,
then find PM.
(a) 7.5 cm (b) 11 cm (c) 9 cm (d) 6 cm
Ans: (b) 11 cm

3. The value of sin 30° cos 60° + sin 60° cos 30° is:
(a) 0 (b) 1 (c) 2 (d) 4
Ans: (b) 1
sin 30° cos 60° + sin 60° cos 30°
1 1  3 3  1 3 1 3 4
           1
2 2  2 2  4 4 4 4

4. Volumes of two spheres are in the ratio 64:27. The ratio of their surface areas is
(a) 3:4 (b) 4:3 (c) 9:16 (d) 16:9
Ans. (d) 16:9

Prepared by: M. S. KumarSwamy, TGT(Maths) Page - 1-


5. A ticket is drawn at random from a bag containing tickets numbered from 1 to 40. The probability
that the selected ticket has a number which is a multiple of 5 is
(a) 1/5 (b) 3/5 (c) 4/5 (d) 1
Ans: (a) 1/5

6. The LCM of two numbers is 182 and their HCF is 13. If one of the numbers is 26, the other
number is
(a) 31 (b) 71 (c) 61 (d) 91
Ans: (d) 91
a × b = HCF (a, b) × LCM (a, b)
⇒ 26 × b = 13 × 182
⇒ b = 13 × 182 / 26 = 91

7. The radii of two concentric circles are 4 cm and 5 cm. The difference in the areas of these two
circles is:
(a) π (b) 7π (c) 9π (d) 13π
Ans. (c) 9π
Required difference = π(52 – 42) = 9π

8. If r = 3 is a root of quadratic equation kr2 – kr – 3 = 0, then the value of k is:


(a) 3/2 (b) 1/2 (c) 2 (d) 5/2
Ans: (b) 1/2
As r = 3 is a root of kr2 – kr – 3 = 0, we have:
9k – 3k – 3 = 0
⇒ 6k − 3 = 0 ⇒ k = 3/6 = 1/2

9. In the below figure, AD = 3 cm, AE = 5 cm, BD = 4 cm, CE = 4 cm, CF = 2 cm, BF = 2.5 cm, then
(a) DE || BC (b) DF || AC (c) EF || AB (d) none of these

Ans: (c) EF || AB

10. If the distance between the points (x, –1) and (3, 2) is 5, then the value of x is
(a) –7 or –1 (b) –7 or 1 (c) 7 or 1 (d) 7 or –1
Ans: (d) 7 or –1

11. Find the value of k so that the following system of equations has no solution:
3x – y – 5 = 0, 6x – 2y + k = 0
(a) 10 (b) -10 (c) 12 (d) -12
Ans: (b) -10
Here a1 = 3, b1 = –1, c1 = –5,
and a2 = 6, b2 = –2, c2 = k.
For no solution,

Prepared by: M. S. KumarSwamy, TGT(Maths) Page - 2-


12. For the following distribution:
Marks Below Below Below Below Below Below
10 20 30 40 50 60
No. of Students 3 12 27 57 75 80
the modal class is
(a) 10 – 20 (b) 20 – 30 (c) 30 – 40 (d) 50 – 60
Ans: (c) 30 – 40

13. If the sum of the areas of two circles with radii R1 and R2 is equal to the area of a circle of radius R,
then:
(a) R1 + R2 = R (b) R12 + R22 = R2 (c) R1 + R2 < R (d) R12 + R22 < R2
Ans. (b) R12 + R22 = R2
According to the given condition,
Area of circle with radius R = Area of circle with radius R1 + Area of circle with radius R2
⇒ πR2 = πR12 + πR22
⇒ R2 = R12 + R22

14. If 2 sin 2θ = √3, then find the value of θ.


(a) 30° (b) 60° (c) 90° (d) 45°
Ans: (a) 30°
2 sin 2θ = √3 ⇒ sin 2θ = √3/2 = sin600
⇒ 2θ = 600 ⇒ θ = 300

15. In figure AT is a tangent to the circle with centre O such that OT = 4 cm and OTA = 30°. Then AT
is equal to

(a) 4 cm (b) 2 cm (c) 2√3 cm (d) 4√3 cm


Ans: (c) 2√3 cm

16. Mode and mean of a data are 12k and 15k. Median of the data is
(a) 12k (b) 14k (c) 15k (d) 16k
Ans: (b) 14k
∵ Mode = 3 median – 2 mean
⇒ 12k = 3 median – 2 × 15k
⇒ 42k = 3 median
⇒ Median = 14k.

17. Which of the following equations has 2 as a root?

Prepared by: M. S. KumarSwamy, TGT(Maths) Page - 3-


(a) x2 – 4x + 5 = 0 (b) x2 + 3x – 12 = 0
(c) 2x2 – 7x + 6 = 0 (d) 3x2 – 6x – 2 = 0
Ans: (c) 2x2 – 7x + 6 = 0

18. If sin A = 1/2, cos B = 1, 0 < A, B ≤ π/2, then the value of cot (A + B) is:
(a) √3 /2 (b) 1/2 (c) 0 (d) √3
Ans: (d) √3
sin A = 1/2 = sin300 ⇒ A = 300
cos B = 1 = cos00 ⇒ B = 00
Now, cot(A+ B) = cot(300 + 00) = cot300 = √3

Direction : In the question number 19 & 20 , A statement of Assertion (A) is followed by a


statement of Reason(R) . Choose the correct option
19. Assertion (A): The value of y is 3, if the distance between the points P(2, -3) and Q(10, y) is 10.
Reason (R): Distance between two points is given by ( x2  x1 )2  ( y2  y1 )2
(a) Both Assertion (A) and Reason (R) are true and Reason (R) is the correct explanation of
Assertion (A)
(b) Both assertion (A) and reason (R) are true and reason (R) is not the correct explanation of
Assertion (A)
(c) Assertion (A) is true but reason(R) is false.
(d) Assertion (A) is false but reason(R) is true.
Ans: (d) Assertion (A) is false but reason(R) is true.

20. Assertion (A): If HCF ( 90, 144) = 18, then LCM (90, 144) = 720
Reason (R): HCF (a, b) x LCM (a, b) = a x b
(a) Both Assertion (A) and Reason (R) are true and Reason (R) is the correct explanation of
Assertion (A).
(b) Both Assertion (A) and Reason (R) are true but Reason (R) is not the correct explanation of
Assertion (A).
(c) Assertion (A) is true but Reason (R) is false.
(d) Assertion (A) is false but Reason (R) is true.
Ans: (a) Both Assertion (A) and Reason (R) are true and Reason (R) is the correct explanation of
Assertion (A).

SECTION-B
Questions 21 to 25 carry 2M each

21. For what value of p will the following pair of linear equations have infinitely many solutions?
(p – 3)x + 3y = p; px + py = 12
Ans: Consider equations (p – 3)x + 3y = p
and px + py = 12
For infinitely many solutions,
p3 3 p
  ...(i)
p p 12
3 p
Consider,   p 2  36  p  6
p 12
p3 3 p
For p = 6, from (i),   is true
p p 12
p3 3 p
For p = – 6, from (i),   is false.
p p 12
Hence, for p = 6, pair of linear equations has infinitely many solutions.

Prepared by: M. S. KumarSwamy, TGT(Maths) Page - 4-


22. In ABC, DE || AB. If AD = 2x, DC = x + 3 , BE = 2x − 1 and CE = x, then find the value of ‘x’
CD CE
Ans:  by BPT
AD BE
x 3 x
⇒ 
2x 2x 1
⇒ 2x2 + 6x – x – 3 = 2x2 ⇒ 5x – 3 = 0 ⇒ x =

23. If sin (A + B) = 1 and sin (A – B) = , 0 ≤ A + B ≤ 90° and A > B, then find A and B.
Ans: sin(A + B) = 1 = sin 90⁰
⇒ A + B = 90⁰ ........(i)
sin (A - B) = 1/2 = sin 30⁰
⇒ A - B = 30⁰ ........(ii)
Solving eq. (i) and (ii), A = 60⁰ and B = 30⁰
OR
1  tan 2 A
Prove that: 2
 tan 2 A
1  cot A
1
2 2
1  tan A sec A cos 2
A sin 2 A
Ans: 2
 2
  2
 tan 2 A
1  cot A cos ec A 1 cos A
2
sin A

24. In the given figure, sectors of two concentric circles of radii 7 cm and 3.5 cm are given. Find the
area of the shaded region. (Use π = )

Ans: Area of the shaded region = area of the sector of 30° with radius 7 cm – area of the sector 30°
with radius 3.5 cm

OR
A horse is placed for grazing inside a rectangular field 70 m by 52 m and is tethered to one corner
by a rope 21 m long. On how much area can it graze?
Ans : Area of the portion that horse can graze = area of the shaded portion.

Prepared by: M. S. KumarSwamy, TGT(Maths) Page - 5-


Shaded portion is a sector of radius 21 m = length of the rope
Angle of this sector = angle of the corners of the rectangle = 90°
Area of the shaded portion that horse can graze

25. In the below figure, ΔABC is circumscribing a circle. Find the length of BC.

Ans: AR = 4 cm
Also, AR = AQ (Length of tangents from A)
⇒ AQ = 4 cm
Now, QC = AC – AQ = 11 cm – 4 cm = 7 cm ...(i)
Also, BP = BR
∴ BP = 3 cm and PC = QC
⇒ PC = 7 cm [From (i)]
Hence, BC = BP + PC = 3 cm + 7 cm = 10 cm

SECTION-C
Questions 26 to 31 carry 3 marks each

26. Given that √3 is irrational, prove that 5 + 2√3 is irrational.


Ans: Let us assume 5 + 2√3 is rational, then it must be in the form of p/q where p and q are co-
prime integers and q ≠0
i.e 5 + 2√3 = p/q
p  5q
So √3 = …(i)
2q
Since p, q, 5 and 2 are integers and q ≠ 0,
RHS of equation (i) is rational.
But LHS of (i) is √3 which is irrational. This is not possible.
This contradiction has arisen due to our wrong assumption that 5 + 2√3 is rational
So, 5 + 2√3 is irrational.

27. Cards numbered 1 to 30 are put in a bag. A card is drawn at random from this bag. Find the
probability that the number on the drawn card is
(i) not divisible by 3.
(ii) a prime number greater than 7.

Prepared by: M. S. KumarSwamy, TGT(Maths) Page - 6-


(iii) not a perfect square number.
Ans : Total possible outcomes of drawing a card from a bag out of 30 cards = 30.
(i) Favourable outcomes for a card numbered not divisible by 3 = 20 (i.e. 1, 2, 4, 5, 7, 8, 10, 11, 13,
14, 16, 17, 19, 20, 22, 23, 25, 26, 28 and 29).
Probability of drawing a card numbered not divisible by 3 = 20/30 = 2/3
(ii) Favourable outcomes for a prime numbered card greater than 7 = 6 (i.e. 11, 13, 17, 19, 23 and
29)
Probability of drawing a prime number card, greater than 7 = 6/30 = 1/5
(iii) Favourable outcomes for not a perfect square numbered card = 25 (leaving 1, 4, 9, 16 and 25)
Probability of drawing a card which is not a perfect square = 25/30 = 5/6

28. A number consists of two digits. Where the number is divided by the sum of its digits, the quotient
is 7. If 27 is subtracted from the number, the digits interchange their places, find the number.
Ans : Let digit at unit place be x, and at tenth place be y.
∴ Number = 10y + x
10 y  x
According to the question,  7 ⇒ 6x – 3y = 0
yx
⇒ 2x – y = 0 ...(i)
Again according to the question, (10y + x) – 27 = 10x + y
⇒ 9x – 9y = – 27 ⇒ x – y = – 3 ...(ii)
Solving for x and y, we get
x = 3 and y = 6
∴ Number is 63.
OR
Students of a class are made to stand in rows. If 4 students are extra in a row, there would be two
rows less. If 4 students are less in a row, there would be four more rows. Find the number of
students in the class.
Ans : Let number of students in a row be x and number of rows be y.
⇒ Total number of students = x . y
From condition 1 : (x + 4) (y – 2) = xy
⇒ xy – 2x + 4y – 8 = xy – 2x + 4y = 8 ...(i)
From condition 2 : (x – 4) (y + 4) = xy xy + 4x – 4y – 16 = xy
⇒ 4x – 4y = 16 ...(ii)
Adding (i) and (ii), we get 2x = 24 ⇒ x = 12
Substituting in (i), we get y = 8.
∴ Total number of students = xy = 12 × 8 = 96.

29. Find the zeroes of the quadratic polynomial 2x2 – x – 6 and verify the relationship between the
zeroes and the coefficients of the polynomial.
Ans: 2x2 – x – 6 = 2x2 – 4x + 3x – 6
= 2x ( x – 2) + 3 ( x – 2 )
x = 2,

Sum of zeroes = 2 + ( )= = = = =

Product of zeroes = 2 x = =

30. Two tangents PA and PB are drawn to a circle with centre O from an external point P. Prove that
APB = 2OAB.

Prepared by: M. S. KumarSwamy, TGT(Maths) Page - 7-


PA & PB are the tangents to a circle, with Centre O from a point P outside it.
PA= PB. (The tangents to a circle from an external point are equal in length )
∠PBA = ∠PAB [Angles opposite to the equal sides of a triangle are equal.]
∠APB+ ∠PBA +∠PAB= 180° [Sum of the angles of a triangle is 180°]
x° + ∠PAB +∠PAB = 180° (∠PBA = ∠PAB)
⇒ x° + 2∠PAB = 180°
⇒ ∠PAB =½(180° - x°)
⇒ ∠PAB =90° - x°/2
⇒ ∠OAB +∠PAB=90°
⇒ ∠OAB =90° - (90° - x°/2)
⇒ ∠OAB = x°/2 ⇒ ∠OAB = ∠APB /2
⇒ ∠APB = 2∠OAB

OR

Prove that opposite sides of a quadrilateral circumscribing a circle subtend supplementary angles at
the centre of the circle.
Ans: Let ABCD be the quadrilateral circumscribing a circle at the center O such that it touches the
circle at the point P,Q,R,S. Let join the vertices of the quadrilateral ABCD to the center of the circle

In ΔOAP and ΔOAS


AP=AS ( Tangents from to same point A)
PO=OS ( Radii of the same circle)
OA=OA ( Common side)
so, ΔOAP=ΔOAS (SSS congruence criterion)
∴ ∠POA=∠AOS (CPCT)
⇒ ∠1=∠8
Similarly, ∠2=∠3, ∠4=∠5 and ∠6=∠7
∠1+∠2 +∠3+∠4+∠5+∠6+∠7+∠8 = 360⁰
⇒ (∠1 +∠8) +(∠2 +∠3) + (∠4 +∠5) + (∠6 +∠7) = 360⁰
⇒ 2(∠1) + 2(∠2) + 2(∠5) + 2(∠6) = 360⁰
⇒ (∠1) + (∠2) + (∠5) + (∠6) = 180⁰
∴ ∠AOD + ∠COD=180⁰
Similarly, ∠BOC + ∠DOA = 180⁰

31. Prove that : sin θ (1 + tan θ) + cos θ (1 + cot θ) = sec θ + cosec θ.


Ans: LHS = sin θ (1 + tan θ) + cos θ (1 + cot θ)
= sin θ + sin θ . tan θ + cos θ + cos θ . cot θ

Prepared by: M. S. KumarSwamy, TGT(Maths) Page - 8-


sin  cos  cos2  sin 2 
= sin θ + sin θ .  cos   cos  .  sin     cos 
cos sin  sin  cos 
sin 2   cos 2  sin 2   cos 2  1 1
=   
sin  cos  sin  cos 

= cosec θ + sec θ = sec θ + cosec θ = RHS

SECTION-D
Questions 32 to 35 carry 5M each

32. If a line is drawn parallel to one side of a triangle, prove that the other two sides are divided in the
same ratio. Using this theorem, find x in below figure, if MN || QR, PM = x cm, MQ = 10 cm, PN
= (x – 2) cm, NR = 6 cm

Ans: Given, To Prove, Constructions and Figure – 1½ marks


Proof – 1½ marks
1 marks to find x = 5 cm

33. A train travels at a certain average speed for a distance of 63 km and then travels at a distance of 72
km at an average speed of 6 km/hr more than its original speed. If it takes 3 hours to complete total
journey, what is the original average speed?
Ans : Let original average speed of the train be x km/h
New average speed be (x + 6) km/h
63
Time taken for a distance of 63 km = hours
x
72
Time taken for a distance of 72 km = hours
x6
63 72 63( x  6)  72 x
According to the question,   3 3
x x6 x( x  6)
63 x  378  72 x 135 x  378
 2
 3 3
x  6x x2  6 x
⇒ 3(x2 + 6x) = 135x + 378 = 3(45x + 126)
⇒ x2 + 6x = 45x + 126 ⇒ x2 + 6x – 45x – 126 = 0 ⇒ x2 – 39x – 126 = 0
2
⇒ x – 42x + 3x – 126 = 0 ⇒ x(x – 42) + 3(x – 42) = 0 ⇒ (x – 42)(x + 3) = 0
∴ x – 42 = 0 or x + 3 = 0
⇒ x = 42 or x = – 3 (rejected)
Therefore, original average speed of the train is 42 km/h.
OR

Prepared by: M. S. KumarSwamy, TGT(Maths) Page - 9-


In a flight of 600 km, an aircraft was slowed due to bad weather. Its average speed for the trip was
reduced by 200 km/hr and time of flight increased by 30 minutes. Find the original duration of
flight.
Ans : Let original speed of the aircraft be x km/hr
Reduced speed = (x – 200) km/hr
600 600 1 600 x  600 x  120000 1
According to given condition,    
x  200 x 2 x( x  200) 2
120000 1
 2  ⇒ x2 – 200x = 240000
x  200 x 2
⇒ x2 – 200x – 240000 = 0
⇒ x2 – 600x + 400x – 240000 = 0
⇒ x(x – 600) + 400(x – 600) = 0
⇒ (x + 400) (x – 600) = 0
⇒ x + 400 = 0 or x – 600 = 0
⇒ x = – 400 (rejected) or x = 600
∴ original speed = 600 km/hr
∴ original duration of flight = = 1 hour

34. The median of the following data is 137. Find the values of x and y, If the total frequency is 68.
Class 105 – 125 – 145 – 165 – 185 –
65 – 85 85 – 105
intervals 125 145 165 185 205
Frequency 4 x 13 20 14 y 4
Ans:
Class Frequency Cumulative Frequency
65-85 4 4
85-105 x 4+x
105-125 13 17+ x
125-145 20 37+ x
145-165 14 51+ x
165-185 y 51 + x + y
185-205 4 55 + x + y
∴ Median class is "125−145." cf = 17 + x , l = 125 , f = 20 h = 20, N = 68
Median = l + ( )xh
⇒ 137 = 125 + ( ) x 20
⇒ 137 – 125 = 17 – x
⇒ x = 17 – 12 = 5 ⇒ 55 + 5 + y = 68 ⇒ y = 8

35. A juice seller serves his customers using a glass as shown in figure. The inner diameter of the
cylindrical glass is 5 cm, but the bottom of the glass has a hemispherical portion raised which
reduces the capacity of the glass. If the height of the glass is 10 cm, find the apparent capacity of
the glass and its actual capacity. [ π = 3.14]

Prepared by: M. S. KumarSwamy, TGT(Maths) Page - 10-


Ans:

OR
A rectangular sheet of paper 30 cm × 18 cm can be transformed into the curved surface of a right
circular cylinder in two ways either by rolling the paper along its length or by rolling it along its
breadth. Find the ratio of the volumes of the two cylinders thus formed.
Ans:

SECTION-E (Case Study Based Questions)


Questions 36 to 38 carry 4M each

36. India is competitive manufacturing location due to the low cost of manpower and strong technical
and engineering capabilities contributing to higher quality production runs. The production of TV
sets in a factory increases uniformly by a fixed number every year. It produced 16000 sets in 6th
year and 22600 in 9th year.

On the basis of the above information, answer any four of the following questions:

Prepared by: M. S. KumarSwamy, TGT(Maths) Page - 11-


(i) What is the production of first year? (1)
(ii) What is the production of 8th year? (1)
(iii) What is the production during first three years? (2)
OR
(iii) In which year, the production is 29,200? (2)
Ans: (i) Rs 5000
(ii) Production during 8th year is (a + 7d) = 5000 + 7(2200) = 20400
(iii) Production during first 3 year = 5000 + 7200 + 9400 = 21600
OR
(iii)an = a + (n – 1)d
⇒ 29200 = 5000 + (n – 1)2200
⇒ (n – 1)2200 = 29200 – 5000 = 24200
⇒ n – 1 = 24200/2200 = 11
⇒ n = 11 + 1 = 12

37. Raj is an electrician in a village. One day power was not there in entire village and villagers called
Raj to repair the fault. After thorough inspection he found an electric fault in one of the electric pole
of height 5 m and he has to repair it. He needs to reach a point 1.3m below the top of the pole to
undertake the repair work.

Based on the above information answer the following questions.


(i) When the ladder is inclined at an angle of α such that √3 tan α + 2 = 5 to the horizontal, find the
angle α. (1)
(ii) In the above situation if BD = 3 cm and BC = 6 cm. Find α (1)
(iii) How far from the foot of the pole should he place the foot of the ladder? (Use √3 = 1.73) (2)
OR
(iii) Given 15 cot α = 8, find sin α. (2)
Ans: (i) tan α + 2 = 5
⇒ tan α = 5 – 2 =3
⇒ tan α = = tan 600
⇒ α = 600
(ii) BD = 3 cm and BC = 6 cm
BD
In ∆BCD, sinα 
BC
3 1
⇒ sinα   = sin300 ⇒ α = 300
6 2
(iii) BD = AD – AC = 5 – 1.3 = 3.7

Prepared by: M. S. KumarSwamy, TGT(Maths) Page - 12-


BD
In ∆BCD, tan600 
DC
3.7
⇒  = 1.73
DC
⇒ DC = 3.7/1.73 = 2.14 m (approx.)

OR
cot α = 8/15
⇒ DC = 8 and BD = 15
From Pythagoras theorem,
BC2 = BD2 + DC2
⇒ BC2 = 152 + 82
⇒ BC = 225 + 64 = 289
⇒ BC = 17
⇒ sin α = BD/BC = 15/17

38. Aditya, Ritesh and Damodar are fast friend since childhood. They always want to sit in a row in the
classroom . But teacher doesn’t allow them and rotate the seats row-wise everyday. Ritesh is very
good in maths and he does distance calculation everyday. He consider the centre of class as origin
and marks their position on a paper in a co-ordinate system. One day Ritesh make the following
diagram of their seating position marked Aditya as A, Ritesh as B and Damodar as C.

(i) What is the distance between A and B ? [1]


(ii) What is the distance between B and C ? [1]
(iii) A point D lies on the line segment between points A and B such that AD :DB = 4 : 3 . What are
the the coordinates of point D ? [2]
OR
(iii) If the point P(k, 0) divides the line segment joining the points A(2, –2) and B(–7, 4) in the ratio
1 : 2, then find the value of k [2]
Ans:

Prepared by: M. S. KumarSwamy, TGT(Maths) Page - 13-


(i) It may be seen easily from figure that coordinates of point A are (- 2, 2).
AB2 = (− 2 + 1)2 + (2 + 2)2 = 1 + 42 = 17
⇒ AB = √17
(ii) It may be seen easily from figure that coordinates of point C are (3, 0).
BC2 = (− 1 − 3)2 + (− 2 − 0)2 = 42 + 4 = 20
⇒ BC = 2√5
m 4
(iii) We have A(- 2, 2) and B(- 1,- 2) and 1 
m2 3
m x  m2 x1 1(4)  3(2) 10
x 1 2  
m1  m2 43 7
m y  m2 y1 2(4)  3(2) 2
y 1 2  
m1  m2 43 7
 10 2 
Coordinates of D is  , 
 7 7 
OR

m1 x2  m2 x1 m y  m2 y1
Using Section Formula, x  and y  1 2 , we get
m1  m2 m1  m2
 2  2  1 7  4  7 3
k    1
 1  2  3 3

Prepared by: M. S. KumarSwamy, TGT(Maths) Page - 14-


KENDRIYA VIDYALAYA GACHIBOWLI, GPRA CAMPUS, HYD-32
SAMPLE PAPER TEST 08 FOR BOARD EXAM 2023
(ANSWERS)
SUBJECT: MATHEMATICS MAX. MARKS : 80
CLASS : X DURATION : 3 HRS
General Instruction:
1. This Question Paper has 5 Sections A-E.
2. Section A has 20 MCQs carrying 1 mark each.
3. Section B has 5 questions carrying 02 marks each.
4. Section C has 6 questions carrying 03 marks each.
5. Section D has 4 questions carrying 05 marks each.
6. Section E has 3 case based integrated units of assessment (04 marks each) with sub-parts of the
values of 1, 1 and 2 marks each respectively.
7. All Questions are compulsory. However, an internal choice in 2 Qs of 5 marks, 2 Qs of 3 marks and
2 Questions of 2 marks has been provided. An internal choice has been provided in the 2marks
questions of Section E
8. Draw neat figures wherever required. Take π =22/7 wherever required if not stated.
SECTION – A
Questions 1 to 20 carry 1 mark each.

1. ABCD is a trapezium with AD ∥ BC and AD = 4cm. If the diagonals AC and BD intersect each
other at O such that AO/OC = DO/OB =1/2, then BC =
(a) 6cm (b) 7cm (c) 8cm (d) 9cm
Ans: (c) 8cm
7
2. In ABC right angled at B, sin A = , then the value of cos C is ………….
25
7 24 7 24
(a) (b) (c) (d)
25 25 24 7
7
Ans: (a)
25
3. Volumes of two spheres are in the ratio 64:27. The ratio of their surface areas is
(a) 3:4 (b) 4:3 (c) 9:16 (d) 16:9
Ans. (d) 16:9
4. A card is selected at random from a well shuffled deck of 52 cards. The probability of its being a
face card is
(a) 3/26 (b) 3/13 (c) 2/13 (d) 1/2
Ans: (b) 3/13
5. If the radii of two circles are in the ratio of 4 : 3, then their areas are in the ratio of :
(a) 4 : 3 (b) 8 : 3 (c) 16 : 9 (d) 9 : 16
Ans: (c) 16 : 9
6. If one zero of the quadratic polynomial x2 + 3x + k is 2, then the value of k is
(a) 10 (b) –10 (c) 5 (d) –5
Ans: (b) –10

7. If two positive integers p and q can be expressed as p = ab2 and q = a3b; a, b being prime numbers,
then LCM (p, q) is
(a) ab (b) a2b2 (c) a3b2 (d) a3b3
Ans: (c) a3b2

Prepared by: M. S. KumarSwamy, TGT(Maths) Page - 1-


8. When 2120 is expressed as the product of its prime factors we get
(a) 2 × 5³ × 53 (b) 2³ × 5 × 53 (c) 5 × 7² × 31 (d) 5² × 7 × 33
Ans: (b) 2³ × 5 × 53

9. In the ∆ABC, D and E are points on side AB and AC respectively such that DE || BC.
If AE = 2 cm, AD = 3 cm and BD = 4.5 cm, then CE equals
(a) 1 cm (b) 2 cm (c) 3 cm (d) 4 cm
Ans: (c) 3 cm

10. The perimeter of a triangle with vertices (0, 4), (0, 0) and (3, 0) is
(a) 5 units (b) 12 units (c) 11 units (d) (7 + √5) units
Ans: (b) 12 units

11. The pair of linear equations 2x + 3y = 5 and 4x + 6y = 10 is


(a) inconsistent (b) consistent (c) dependent consistent (d) none of these
Ans: (c) dependent consistent

12. The median class of the following data is:


Marks 0 – 10 10 – 20 20 – 30 30 – 40 40 – 50 50 – 60
No. of students 8 10 12 22 30 18
(a) 20 – 30 (b) 30 – 40 (c) 40 – 50 (d) 50 – 60
Ans: (b) 30 – 40
13. The area of a quadrant of a circle, whose circumference is 22 cm, is
11 77 77 77
(a) cm2 (b) cm2 (c) cm2 (d) cm2
8 8 2 4
77
Ans: (b) cm2
8
14. In ΔABC, right angled at B, AB = 5 cm and sin C = 1/2. Determine the length of side AC.
(a) 10 cm (b) 15 cm (c) 20 cm (d) none of these
Ans: (a) 10 cm
15. If two tangents inclined at an angle of 60ᵒ are drawn to a circle of radius 3cm, then the length of
each tangent is equal to
3 3
(a) cm (b) 3 cm (c) 6 cm (d) 3 3
2
Ans: (d) 3 3

16. If the mean of a frequency distribution is 8.1 and fi = 20, fixi = 132 + 5k, then k =
(a) 3 (b) 4 (c) 5 (d) 6
Ans: (d) 6

17. If x2 + k (4x + k -1) + 2 = 0 has equal roots, then k = ………


2 2 3 1 3 1
(a)  ,1 (b) , 1 (c) , (d) ,
3 3 2 3 2 3
2
Ans: (b) , 1
3
5sin   3cos 
18. If 5 tan θ = 4, then the value of is
5sin   2 cos 
(a) 1/6 (b) 1/7 (c) 1/4 (d) 1/5
Ans: (a) 1/6

Prepared by: M. S. KumarSwamy, TGT(Maths) Page - 2-


Direction : In the question number 19 & 20 , A statement of Assertion (A) is followed by a
statement of Reason(R) . Choose the correct option

19. Statement A (Assertion): If product of two numbers is 5780 and their HCF is 17, then their LCM
is 340
Statement R( Reason) : HCF is always a factor of LCM
(a) Both Assertion (A) and Reason (R) are true and Reason (R) is the correct explanation of
Assertion (A).
(b) Both Assertion (A) and Reason (R) are true but Reason (R) is not the correct explanation of
Assertion (A).
(c) Assertion (A) is true but Reason (R) is false.
(d) Assertion (A) is false but Reason (R) is true.
Ans: (b) Both Assertion (A) and Reason (R) are true but Reason (R) is not the correct explanation
of Assertion (A).

20. Assertion (A): The value of y is 3, if the distance between the points P(2, -3) and Q(10, y) is 10.
Reason (R): Distance between two points is given by ( x2  x1 )2  ( y2  y1 )2
(a) Both Assertion (A) and Reason (R) are true and Reason (R) is the correct explanation of
Assertion (A)
(b) Both assertion (A) and reason (R) are true and reason (R) is not the correct explanation of
Assertion (A)
(c) Assertion (A) is true but reason(R) is false.
(d) Assertion (A) is false but reason(R) is true.
Ans: (a) Both Assertion (A) and Reason (R) are true and Reason (R) is the correct explanation of
Assertion (A)

SECTION-B
Questions 21 to 25 carry 2M each

21. For what values of k will the following pair of linear equations have infinitely many solutions? kx
+ 3y – (k – 3) = 0 and 12x + ky – k = 0
Ans: Comparing with a1 x  b1 y  c1 and a2 x  b2 y  c2
a1  k , a2  12, b1  3, b2  k , c1  k  3, c2  k
a b c
For infinite solutions, 1  1  1
a2 b2 c2
k 3 k 3 k 3
      k 2  36  k  6
12 k k 12 k

22. In the given figure below, AD/AE=AC/BD and ∠1=∠2. Show that Δ BAE~ ΔCAD .

Ans: In ΔABC, ∠1 = ∠2
∴ AB = BD ………(i)
AD AC
Given, 
AE BD

Prepared by: M. S. KumarSwamy, TGT(Maths) Page - 3-


AD AC
Using equation (i), we get  ……….(ii)
AE AB
AC AD
In ΔBAE and ΔCAD, by equation (ii), 
AB AE
and ∠A= ∠A (common)
∴ ΔBAE ~ ΔCAD [By SAS similarity criterion]

1
23. If sin (A + B) = √3/2 and sin (A – B) = , 0 ≤ A + B ≤ 90° and A > B, then find A and B.
2
Ans: sin(A + B) = √3/2 = sin 60⁰
⇒ A + B = 60⁰ ........(i)
sin (A - B) = 1/2 = sin 30⁰
⇒ A - B = 30⁰ ........(ii)
Solving eq. (i) and (ii), A = 45⁰ and B = 15⁰
OR
(1  sin  )(1  sin  )
If tan θ =3/4, evaluate
(1  cos  )(1  cos  )
3 4
Ans: tan    cot  
4 3
2
(1  sin  )(1  sin  ) 1  sin 2  cos 2  2  4  16
   cot     
(1  cos  )(1  cos  ) 1  cos2  sin 2  3 9
22
24. If the perimeter of a protractor is 72 cm, calculate its area. (Use π = )
7
36r 36r
Ans: ( r  2r )    72  r  14
7 7
 r 2 11
Now area of the protractor is  14 14  22  14  308cm 2
2 7
OR
Two circular pieces of equal radii and maximum area, touching each other are cut out from a
Rectangular card board of dimensions 14 cm × 7 cm. Find the area of the remaining card board.
[Use π = 22/7 ]
Ans: Here, r = 7/2 cm , l = 14 cm, b = 7 cm
Area of the remaining card board
= area of rectangle – 2 ( area of circle)
= l x b – 2 r2
= 14 x 7 – 2 x x x = 98 – 77 = 21 cm2

25. Two concentric circles are of radii 5 cm and 3 cm. Find the length of the chord of the larger circle
which touches the smaller circle.
Ans: Let O be the centre of the concentric circle of radii 5 cm and 3 cm respectively. Let AB be a
chord of the larger circle touching the smaller circle at P

Prepared by: M. S. KumarSwamy, TGT(Maths) Page - 4-


Then AP = PB and OP⊥AB
Applying Pythagoras theorem in △OPA, we have
OA2 = OP2 + AP2 ⇒ 25 = 9 + AP2
⇒ AP2 = 16 ⇒ AP = 4 cm
∴ AB = 2AP = 8 cm

SECTION-C
Questions 26 to 31 carry 3 marks each

26. Given that √3 is irrational, prove that 2 + 5√3 is irrational.


Ans: Let us assume 2 + 5√3 is rational, then it must be in the form of p/q where p and q are co-
prime integers and q ≠0
i.e 2 + 5√3 = p/q
p  2q
So √3 = …(i)
5q
Since p, q, 5 and 2 are integers and q ≠ 0,
RHS of equation (i) is rational.
But LHS of (i) is √3 which is irrational. This is not possible.
This contradiction has arisen due to our wrong assumption that 2 + 5√3 is rational
So, 2 + 5√3 is irrational.

27. Two dice are thrown at the same time. What is the probability that the sum of the two numbers
appearing on the top of the dice is (i) 7? (ii) 14? (iii) equal to 12?
Ans: (i) P(sum of the numbers is 7) = 6/36 = 1/6
(ii) P(sum of the numbers is 14) = 0/36 = 0
(iii) P(sum of the numbers is 12) = 1/36

28. The area of a rectangle gets reduced by 9 square units, if its length is reduced by 5 units and breadth
is increased by 3 units. If we increase the length by 3 units and the breadth by 2 units, the area
increases by 67 square units. Find the dimensions of the rectangle.
Ans: Let length and breadth be x and y , Area = xy
1st condition: ( x – 5 ) ( y + 3 ) = xy – 9
⇒ 3x – 5y = 6
2nd condition: ( x + 3 ) ( y + 2 ) = xy + 67
⇒ 2x + 3y = 61
Solve 1st and 2nd equations, we get x = 17 and y = 9
Hence, Length of rectangle = 17 units and breadth of rectangle = 9 units
OR
A train covered a certain distance at a uniform speed. If the train would have been 6 km/h faster, it
would have taken 4 hours less than the scheduled time. And, if the train were slower by 6 km/hr; it
would have taken 6 hours more than the scheduled time. Find the length of the journey.
Ans: Let the actual speed of the train be x km/hr and let the actual time taken be y hours.
Distance covered is xy km If the speed is increased by 6 km/hr, then time of journey is reduced by 4
hours i.e., when speed is (x+6)km/hr, time of journey is (y−4) hours.
∴ Distance covered = (x + 6)(y − 4)
⇒ xy = (x + 6)(y − 4) ⇒ −4x + 6y − 24 = 0 ⇒ −2x + 3y −12 = 0 ………….(i)
Similarly xy = (x − 6)(y + 6) ⇒ 6x − 6y − 36 = 0 ⇒ x − y − 6 = 0 …………(ii)
Solving (i) and (ii) we get x=30 and y=24
Putting the values of x and y in equation (i), we obtain
Distance = (30 × 24)km =720km.
Hence, the length of the journey is 720km.

Prepared by: M. S. KumarSwamy, TGT(Maths) Page - 5-


29. Find the zeroes of the quadratic polynomial 6x2– 3 – 7x and verify the relationship between the
zeroes and the coefficients of the polynomial.
Ans: 6x2 – 7x – 3 = 0
6x2 – 9x + 2x – 3 = 0
3x(2x – 3) + 1(2x – 3) = 0
 (3x + 1) (2x – 3) = 0
1 3
x = ,
3 2
1 3 2  9 7 b 7 b
Now,        and     
3 2 6 6 a 6 a
1 3 1 c 1 c
    and    
3 2 2 a 2 a

30. In the below figure, XY and X′Y′ are two parallel tangents to a circle with centre O and another
tangent AB with point of contact C intersecting XY at A and X′Y′ at B. Prove that ∠AOB = 90°.

Ans: Join OC. Since, the tangents drawn to a circle from an external point are equal.
∴ AP = AC

In Δ PAO and Δ AOC, we have:


AO = AO [Common]
OP = OC [Radii of the same circle]
AP = AC
⇒ Δ PAO ≅ Δ AOC [SSS Congruency]
∴ ∠PAO = ∠CAO = ∠1
∠PAC = 2 ∠1 ...(1)
Similarly ∠CBQ = 2 ∠2 ...(2)
Again, we know that sum of internal angles on the same side of a transversal is 180°.
∴ ∠PAC + ∠CBQ = 180°
⇒ 2 ∠1 + 2 ∠2 = 180° [From (1) and (2)]
⇒ ∠1 + ∠2 = 180°/2 = 90° ...(3)
Also ∠1 + ∠2 + ∠AOB = 180° [Sum of angles of a triangle]
⇒ 90° + ∠AOB = 180°
⇒ ∠AOB = 180° − 90° ⇒ ∠AOB = 90°.

Prepared by: M. S. KumarSwamy, TGT(Maths) Page - 6-


OR
Prove that opposite sides of a quadrilateral circumscribing a circle subtend supplementary angles at
the centre of the circle.
Ans: Let ABCD be the quadrilateral circumscribing a circle at the center O such that it touches the
circle at the point P,Q,R,S. Let join the vertices of the quadrilateral ABCD to the center of the circle

In ΔOAP and ΔOAS


AP=AS ( Tangents from to same point A)
PO=OS ( Radii of the same circle)
OA=OA ( Common side)
so, ΔOAP=ΔOAS (SSS congruence criterion)
∴ ∠POA=∠AOS (CPCT)
⇒ ∠1=∠8
Similarly, ∠2=∠3, ∠4=∠5 and ∠6=∠7
∠1+∠2 +∠3+∠4+∠5+∠6+∠7+∠8 = 360⁰
⇒ (∠1 +∠8) +(∠2 +∠3) + (∠4 +∠5) + (∠6 +∠7) = 360⁰
⇒ 2(∠1) + 2(∠2) + 2(∠5) + 2(∠6) = 360⁰
⇒ (∠1) + (∠2) + (∠5) + (∠6) = 180⁰
∴ ∠AOD + ∠COD=180⁰
Similarly, ∠BOC + ∠DOA = 180⁰

sin   cos   1
31. Prove that  sec   tan 
sin   cos   1
tan   1  sec 
Ans: LHS = tan   1  sec  (Dividing numerator and denominator by cos )

tan   sec   1

tan   1  sec 
tan   sec   (sec 2   tan 2  )

tan   1  sec 
(sec   tan  )(1  sec   tan  )

tan   1  sec 
 sec   tan  = RHS

SECTION-D
Questions 32 to 35 carry 5M each

32. Prove that if a line is drawn parallel to one side of a triangle intersecting the other two sides in
distinct points, then the other two sides are divided in the same ratio.
Using the above theorem prove that a line through the point of intersection of the diagonals and
parallel to the base of the trapezium divides the non parallel sides in the same ratio.
Ans: For the Theorem :
Given, To prove, Construction and figure of 1½ marks
Proof of 1½ marks

Prepared by: M. S. KumarSwamy, TGT(Maths) Page - 7-


Let ABCD be a trapezium DC ∥ AB and EF is a line parallel to AB and hence to DC.
Join AC, meeting EF in G.

33. A motor boat whose speed is 18 km/h in still water takes 1 hour more to go 24 km upstream than to
return downstream to the same spot. Find the speed of the stream.
Ans: Let the speed of the stream be x km/h.
Therefore, the speed of the boat upstream = (18 – x) km/h and the speed of the boat downstream =
(18 + x) km/h.
24
The time taken to go upstream = distance/speed =
18  x
24
Similarly, the time taken to go downstream =
18  x
24 24
According to the question,  1
18  x 18  x
24(18 + x) – 24(18 – x) = (18 – x) (18 + x)
x2 + 48x – 324 = 0
x = 6 or – 54
Since x is the speed of the stream, it cannot be negative. So, we ignore the root x = – 54. Therefore,
x = 6 gives the speed of the stream as 6 km/h.

OR
An express train takes 1 hour less than a passenger train to travel 132 km between Mysore and
Bangalore (without taking into consideration the time they stop at intermediate stations). If the
average speed of the express train is 11km/h more than that of the passenger train, find the average
speed of the two trains.
Ans: Ans: Let the average speed of the passenger train = x km/hour
And the average speed of the express train = (x + 11) km/hour.
132
The time taken by the passenger train = hour
x
132
and the time taken by the express train = hour
x  11
132 132
According to the question,  1
x  11 x
⇒ x2 + 11x −1452 = 0
⇒ (x − 33)(x + 44) = 0 ⇒ x = 33, x = − 44
The speed cannot be negative, so the speed of the passenger train is 33 km/hour and the speed of the express
train is 33 + 11 = 44 km/ hour.

34. A life insurance agent found the following data for distribution of ages of 100 policy holders.
Calculate the median age, if policies are given only to persons having age 18 years onwards but
less than 60 years.
Age (in years) Number of policy holders

Prepared by: M. S. KumarSwamy, TGT(Maths) Page - 8-


Below 20 2
20 – 25 4
25 – 30 18
30 – 35 21
35 – 40 33
40 – 45 11
45 – 50 3
50 – 55 6
55 – 60 2
Ans:

35. Ramesh made a bird-bath for his garden in the shape of a cylinder with a hemispherical depression
at one end. The height of the cylinder is 1.45 m and its radius is 30 cm. Find the total surface area
of the bird-bath.

Ans: Let h be height of the cylinder, and r the common radius of the cylinder and hemisphere.
Then, the total surface area = CSA of cylinder + CSA of hemisphere
= 2 rh + 2 r2 = 2 r (h + r)

Prepared by: M. S. KumarSwamy, TGT(Maths) Page - 9-


22
=2x x 30 (145 + 30) cm2
7
22
=2x x 30 x 175 cm2
7
= 33000 cm2 = 3.3 m2
OR
A tent is in shape of a cylinder surmounted by a conical top. If the height and diameter of the
cylindrical part are 2.1m and 4m respectively and the slant height of the top is 2.8m. Find the area
of canvas used for making the tent. Also find the cost of canvas of the tent at the rate of 500 per m2.
Ans: Radius = 2m, Slant height l= 2.8m , height h= 2.1m
Cost of canvas per m2= Rs.500
Area of canvas used = CSA of cone + CSA of cylinder
= πrl + 2πrh
=22/7 x 2 x 2.8 + 2 x 22/7 x 2 x 2.1
=17.6 + 26.4
=44m2
Cost of the canvas of tent =44 x 500
=Rs.22,000

SECTION-E (Case Study Based Questions)


Questions 36 to 38 carry 4M each

36. In the month of April to June 2022, the exports of passenger cars from India increased by 26% in
the corresponding quarter of 2021–22, as per a report. A car manufacturing company planned to
produce 1800 cars in 4th year and 2600 cars in 8th year. Assuming that the production increases
uniformly by a fixed number every year.

Based on the above information answer the following questions.


(i) Find the production in the 1st year. (1)
(ii) Find the production in the 12th year. (1)
(iii) Find the total production in first 10 years. (2)
OR
(iii) In how many years will the total production reach 31200 cars? (2)
Ans: (i) Since the production increases uniformly by a fixed number every year, the number of Cars
manufactured in 1st, 2nd, 3rd, . . .,years will form an AP.
So, a + 3d = 1800 & a + 7d = 2600
So d = 200 & a = 1200
(ii) a12 = a + 11d ⇒ a30 = 1200 + 11 × 200
⇒ a12 = 3400
n 10
(iii) S n  [2a  (n  1) d ]  S10  [2  1200  (10  1)  200]
2 2

Prepared by: M. S. KumarSwamy, TGT(Maths) Page - 10-


 S10  5[2400  1800]  5  4200  21000
OR
n
S n  [2a  (n  1) d ]  31200
2
n
 [2  1200  ( n  1)  200]  31200
2
n
  200[12  ( n  1)]  31200
2
⇒ n[12 + (n − 1) ] = 312
⇒ n2 + 11n – 312 = 0
⇒ n2 + 24n – 13n – 312 = 0
⇒ (n +24)(n – 13) = 0
⇒ n = 13 or – 24.
As n can’t be negative. So n = 13

37. A 1.2 m tall girl spots a balloon moving with the wind in a horizontal line at a height of 88.2 m
from the ground. The angle of elevation of the balloon from the eyes of the girl at any instant is
60°. After 30 seconds, the angle of elevation reduces to 30° (see the below figure).

Based on the above information, answer the following questions. (Take √3 =1.732)
(i) Find the distance travelled by the balloon during the interval. (2)
(ii) Find the speed of the balloon. (2)
OR
(ii) If the elevation of the sun at a given time is 30°, then find the length of the shadow cast by a
tower of 150 feet height at that time. (2)
Ans: (i) In the figure, let C be the position of the observer (the girl).
A and P are two positions of the balloon.
CD is the horizontal line from the eyes of the (observer) girl.
Here PD = AB = 88.2 m − 1.2 m = 87 m

Prepared by: M. S. KumarSwamy, TGT(Maths) Page - 11-


Thus, the required distance between the two positions of the balloon = 58 √3 m
= 58 x 1.732 = 100.46 m (approx.)
(ii) Speed of the balloon = Distance/time = 100.46/30 = 3.35 m/s (approx.)

OR

In right ∆ABC
AB
= tan 30°
BC
150 1
   BC  150 3 feet
BC 3

38. Resident Welfare Association (RWA) of a Gulmohar Society in Delhi have installed three electric
poles A, B and C in a society’s common park. Despite these three poles, some parts of the park are
still in dark. So, RWA decides to have one more electric pole D in the park. The park can be
modelled as a coordinate systems given below.

On the basis of the above information, answer any four of the following questions:
(i) What is the position of the pole C? (1)

Prepared by: M. S. KumarSwamy, TGT(Maths) Page - 12-


(ii) What is the distance of the pole B from the corner O of the park? (1)
(iii) Find the position of the fourth pole D so that four points A, B, C and D form a parallelogram.
(2)
OR
(iii) What is the distance between poles A and C? (2)
Ans: (i) From the given diagram we can easily get that position of the pole C (5, 4).
(ii) Coordinates of B is (6, 6).
Distance from origin = (6  0) 2  (6  0) 2  36  36  72  6 2
(iii) If ABCD is a parallelogram, the diagonals bisects each other. Here AC and BD are diagonals.
 25 7 4
Mid-point of AC =  ,  = (3.5, 5.5)
 2 2 
Now, mid-point of diagonal, BD will be (3.5, 5.5) also.
Let, the coordinates of D be (x, y)
6 x 6 y
Now,  3.5,  5.5,
2 2
x = 1 and y = 5
OR
(iii) Coordinates of A are (2, 7) and coordinates of C are (5, 4).
Distance between pole A and C ,
AC = (5  2) 2  (4  7) 2  9  9  18  3 2

Prepared by: M. S. KumarSwamy, TGT(Maths) Page - 13-

You might also like